✅ DDx: Chest Pain, 🛫Dyspnea, Syncope, 🔥 IE, ECG 📈 Flashcards

1
Q

Differential Diagnosis of Chest Pain:

A

Skin: Laceration, Burns, Herpes

Subq: Cellulitis, Abscess

MSK: Chostochondritis, sprains, strains, myositis

Pleural Space: Pleurisy, Pulmonary embolism, Pulmonary Tumor, Pneumothorax, Pneumonia

Pericardium: Pericarditis

Heart: Acute Coronary Syndrome, MI, myositis

Esophagus: Esophageal rupture, GERD, Esophagitis, Boerhaves

Trachea: Tracheitis, Tracheal Tear

Aorta: Aortic dissection, Aortic stenosis

Drugs: Cocaine

Psychiatric: Panic Disorder

How well did you know this?
1
Not at all
2
3
4
5
Perfectly
2
Q

Classic Angina

A

Classic:

  • Typical location (eg, 🗡substernal), quality & duration
  • Provoked by exercise OR emotional stress
  • Relieved by rest OR nitroglycerin

Atypical

  • 2 of the 3 characteristics of classic angina

Nonanginal

  • <2 of the 3 characteristics of classic angina
How well did you know this?
1
Not at all
2
3
4
5
Perfectly
3
Q

Atypical Angina

A

Pain that has the quality and characteristics of angina, OR occurs with exertion, but NOT both:

May be a sense of heaviness not consistently related to exertion or relieved by rest, or it may be pain with an atypical character (sharp or stabbing) but predictably brought on by exercise and relieved by rest.

How well did you know this?
1
Not at all
2
3
4
5
Perfectly
4
Q

Vasospastic angina (formerly Prinzmetal angina)

A

Pathogenesis

  • Hyperreactivity of coronary smooth muscle.
    • Caused by severe spasm of an epicardial coronary artery. The area of vasospasm is often near a nonhemodynamically significant atherosclerotic lesion.

Clinical presentation

  • Young patients (age <50)
  • Smoking (minimal other CAD risk factors)
  • Recurrent chest discomfort
    • Occurs at rest or during sleep
    • Spontaneous resolution <15 minutes

Diagnosis

  • Ambulatory ECG: ST elevation
  • Coronary angiography: No CAD

Treatment

  • 🍦Calcium channel blocker (preventive)
  • Sublingual nitroglycerin (abortive)

Vascular smooth muscle hyperreactivity leads to focal or diffuse spasm of the coronary arteries, transient myocardial ischemia, and resulting angina. Patients typically have recurrent episodes of chest discomfort that occur at rest or during sleep. Cigarette smoking is a known risk factor, but patients are typically young and lack other risk factors for coronary artery disease (eg, hypertension, diabetes). As the vasospasm leads to transmural myocardial ischemia, the diagnosis is typically made by ambulatory ECG showing contiguous ST elevation during an episode of chest discomfort.

The underlying pathophysiologic mechanism of vasospastic angina is similar to that of Raynaud phenomenon, a disorder characterized by cold- or stress-induced hyperreactivity of the digital arterial smooth muscle, leading to episodic vasospasm in the fingers and toes. Accordingly, calcium channel blockers are an effective first-line pharmacologic therapy for both Raynaud phenomenon and vasospastic angina. However, despite the underlying pathophysiologic similarities, an increased prevalence of Raynaud phenomenon in patients with vasospastic angina (or vice versa) has not been clearly established.

How well did you know this?
1
Not at all
2
3
4
5
Perfectly
5
Q

Stress Testing🏃🏽‍♀️🧪 Rx: 💊

A

Dx:

Exercise ECG 🏃🏽‍♀️ testing is the standard stress test for the diagnosis of CAD in patients with normal baseline ECG findings. Stress testing is most useful in patients with an intermediate pretest probability of CAD. For patients with a low pretest probability of CAD, stress testing is not useful because an abnormal test result is likely a false-positive finding and a normal test result only confirms the low pretest probability of CAD. For patients with a high pretest probability of CAD, stress testing is not useful to diagnose CAD and empiric medical therapy should be initiated.

Cx:abnormalities that limit ST-segment analysis:(left bundle branch block, left ventricular hypertrophy, paced rhythm, Wolff-Parkinson-White pattern), imaging:

🔊echocardiography (TTE) or ☢ nuclear [Thallium, dipyridamole] stress test increases diagnostic accuracy and ability to determine the site and extent of ischemia. For women in their mid 40’s, stress ECGs are often false positive, so a stress test with imaging is most appropriate. Cx: Claudication

Pharmacologic stimulation of heart rate should be used in patients who are unable to exercise. Cx: COPD (Adenosine)

🎵 Adenosine and its synthetic analogs (eg, regadenoson, apadenoson) stimulate adenosine A2A receptors on vascular smooth muscle cells, causing coronary vasodilation and increased myocardial blood flow. There is a several-fold augmentation of blood flow in nonobstructed coronary arteries. Blood flow is increased in stenosed coronary arteries as well but to a much lesser extent. This relative blood flow difference is magnified from rest, causing a detectable reduction in radioactive isotope uptake by myocardial cells in areas supplied by a stenotic coronary artery (appears as an ischemic defect on myocardial perfusion imaging).

Coronary angiography allows direct evaluation of the coronary anatomy, with possible percutaneous coronary intervention or surgical revascularization if indicated.

Rx:

β-Blockers

Used in patients with a hx of MI and in stable heart failure. The dose should be adjusted to achieve a heart rate of 50 to 60 beats/min. [Complete β-blockade typically results in a resting pulse rate of approximately 55 to 60/min.]

🍦 Dihydropyridine calcium channel blockers (eg, amlodipine, felodipine, nifedipine)

[Second-line agents] Reduce blood pressure; do not affect heart rate and can be used with β-blockers. Avoid short-acting agents (such as nifedipine).

🍦 Nondihydropyridine calcium channel blockers (verapamil, diltiazem)

Mostly used in patients who cannot take β-blockers. Avoid in patients with heart failure; use with caution in patients taking β-blockers (bradycardia).

🃏 ACE inhibitors

Reduce blood pressure and afterload by a reduction in peripheral vascular resistance. Reduce ventricular remodeling and fibrosis after infarction. Improve long-term survival in patients with LVEF ≤40% and, possibly, in patients with high cardiovascular risk (eg, diabetes mellitus, PVD).

💣Long-acting nitrates

Can be used with β-blockers and calcium channel blockers. Tachyphylaxis occurs with continued use; requires nitrate-free period (8-12 h/d). Side effects include headache. Avoid in patients taking PDE-5 inhibitors.

💣Short-acting nitrates

Dilate coronary arteries and reduce preload. Indicated for all patients with chronic stable angina for use on an as-needed basis.

👨🏽Ranolazine

Indicated as add-on therapy for patients not responding to standard therapy; used in combination with a nitrate, β-blocker, or calcium channel blocker. Avoid using with verapamil or diltiazem (prolongs QT interval).

🧯 Aspirin

Indicated for all patients with stable angina, barring contraindications; reduces major cardiovascular events by 33%.

♨ Thienopyridine derivatives (eg, clopidogrel, ticlopidine, prasugrel)

Aspirin alternatives, but significantly more expensive. Improve outcomes in patients with recent ACS or stent placement. In patients with stable CAD, thienopyridine derivatives do not improve outcomes.

Statins

In patients with mild to moderate elevations in total and LDL cholesterol and a history of MI, statins are associated with a 24% risk reduction for fatal and nonfatal MI.

Tx: Coronary revascularization has been shown to be beneficial in patients with chronic stable angina and the following conditions: angina pectoris that is refractory to medical therapy; a large area of ischemic myocardium and high-risk criteria on stress testing; high-risk coronary anatomy, including left main coronary artery stenosis or three-vessel disease; and significant coronary artery disease with reduced left ventricular systolic function. In appropriately selected patients, revascularization, with either percutaneous coronary intervention or coronary artery bypass grafting (CABG) surgery, has been shown to reduce angina, increase longevity, and improve left ventricular performance.

How well did you know this?
1
Not at all
2
3
4
5
Perfectly
6
Q

Pulmonary embolism (PE)

A

Acute-onset dyspnea and pleuritic chest pain (exacerbated by deep breathing, coughing, sneezing, or laughing🤣) are the most common symptoms, occurring in 73% and 66% of patients, respectively.

Tachypnea (70% of cases), tachycardia (30%), and low-grade fever (15%)

Sudden-onset dyspnea, nonproductive cough, tachycardia, and mild hypoxia is highly suggestive of acute pulmonary embolism (PE).

Lower extremity deep vein thrombosis (DVT) is divided into 2 categories:

  • Proximal/thigh (eg, iliac, femoral, popliteal): These are the source of >90% of acute PEs, probably due to their large caliber and proximity to the lungs.
  • Distal/calf: These are less likely to embolize and more likely to spontaneously resolve .

The Wells score, has been established to help the clinician assess the likelihood of DVT.

Modified Wells score

+3 points

Clinical signs of DVT: Paralysis or recent plaster cast, recent immobilization or major surgery, tenderness along the deep veins, swelling of the entire leg, a difference in calf circumference of more than 3 cm compared with the other leg, pitting edema, and collateral superficial veins.

Alternate diagnosis less likely than PE: Clinical suspicion that an alternative diagnosis is likely is assigned -2 points.

+1.5 points

Previous PE or DVT

Heart rate >100

Recent surgery or immobilization

+1 point

Hemoptysis

Cancer

Total score for clinical probability
≤4 = PE unlikely
>4 = PE likely

Dx: A D-dimer assay is a simple, relatively noninvasive test that has been shown to have a high negative predictive value, especially if suspicion for DVT is low.

If intermediate or high probability, ventilation-perfusion scan or spiral CT is indicated.

Given clinically stablity (normotensive, mild hypoxemia) with no evidence of distress, the diagnosis of PE can be confirmed with CT angiography (CTA). If CTA confirms PE, clinical judgment can dictate whether anticoagulation is initiated or other options are pursued (eg, inferior vena cava filter placement) based on the estimated risk of bleeding from the peptic ulcer.

A ventilation-perfusion scan is the most appropriate study to confirm the suspected diagnosis of pulmonary embolism in this patient with kidney failure. Ventilation-perfusion scans detect abnormalities of blood flow in comparison to the pattern of ventilation, with areas of mismatch between perfusion and ventilation being evidence of vascular occlusion due to a pulmonary embolus.

CRX shows tachycardia (only 10% S1Q3T3)

📉 RH Cath: Low or normal pulmonary capillary wedge pressure is expected in acute pulmonary embolism due to impaired blood flow through the pulmonary circulation to the left atrium.

Tx: Early, effective anticoagulation decreases the mortality risk of acute PE and should be considered in patients without absolute contraindications (eg, hemorrhagic stroke, massive gastrointestinal bleed).

Intravenous or subcutaneous unfractionated heparin, low-molecular-weight heparin, or fondaparinux. Most patients with pulmonary embolism are treated in the hospital, although carefully selected, stable patients may be candidates for outpatient treatment. Following initial therapy, patients are usually transitioned to warfarin for long-term therapy, with factor Xa and direct thrombin inhibitors being increasingly-available options for this purpose. UFH is primarily cleared by the reticuloendothelial system rather than the kidneys (CKD patients). Fondaparinux is cleared exclusively by the kidneys. Therefore, it is contraindicated in patients with poor renal function. In addition, fondaparinux is not reversible with protamine. Consequently, potential bleeding is much more difficult to treat.

Bridge: 🖐🏽5 days of overlap with LMWH and warfarin therapy and an international normalized ratio of 2 or more for 24 hours. Randomized clinical trials show that 5 to 7 days of treatment with unfractionated heparin is as effective as 10 to 14 days of treatment when transitioning to warfarin therapy. If a patient is receiving an adequate warfarin dose, it takes at least 5 days for vitamin K-dependent factor activity levels to decrease sufficiently for therapeutic anticoagulation (INR of 2-3) to occur.

How well did you know this?
1
Not at all
2
3
4
5
Perfectly
7
Q

Pneumothorax

A

Sudden onset of pleuritic chest pain and dyspnea in a smoker or COPD patient. Other findings include sudden, sharp, nonradiating pleuritic chest pain and shortness of breath with hyperresonance, decreased breath sounds, and decreased chest wall expansion on the side of the pneumothorax in a patient with underlying lung disease.

Dx: Chest radiograph (initial test of choice) or CT scan confirms the diagnosis.

Pneumothorax occurring in patients without known lung disease or a clear precipitating cause is termed primary spontaneous pneumothorax (PSP). PSP tends to occur more often in men, smokers, and those with a family history of PSP.

How well did you know this?
1
Not at all
2
3
4
5
Perfectly
8
Q

Aortic Dissection

A

Clinical features

  • History of HTN, Marfan syndrome, cocaine use, turner syndrome (bicuspid aortic valve, aortic root dilation, aortic coarctation, and hypertension), pregnancy (eg, increased blood volume).
  • Severe, sharp, tearing chest or back pain
  • ± >20 mm Hg variation in SBP between arms

Diagnosis

  • ECG: normal or nonspecific ST- & T-wave changes
  • Chest x-ray: mediastinal widening
  • CT angiography or TEE for definitive diagnosis

Complications due to extension (involved structure)

  • Stroke (carotid artery)
  • Acute aortic regurgitation (aortic root/valve)
  • Horner syndrome (carotid sympathetic plexus)
  • Myocardial ischemia/infarction (coronary artery ostia)
  • Pericardial effusion/tamponade (pericardium)
  • Hemothorax (pleural cavity)
  • Renal injury (renal arteries)
  • Abdominal pain (mesenteric arteries)
  • Lower extremity paraplegia (spinal arteries)

Treatment

  • Pain control (eg, morphine)
  • Intravenous beta blockers (eg, esmolol)
  • ± Sodium nitroprusside (if SBP >120 mm Hg)
  • Emergent surgical repair for ascending (type A) dissection

TAA usually results from age-related degenerative changes that lead to disruption of the aortic wall medial layer with loss of elasticity and consequent aortic dilation. The changes are likely due to a combination of enzymatic breakdown of structural proteins (eg, collagen, elastin) and physical factors such as systemic hypertension and repeated stress from the pulsating arterial wave. Underlying connective tissue disease (Marfans; cystic medial necrosis (described in patients with bicuspid aortic valves), syphilis, Ehlers-Danlos syndrome, trauma, and bacterial infections also increases the risk. Although Marfan syndrome is responsible for almost 50% of the aortic dissections seen in patients age <40, it is an uncommon cause in older patients (age >60).

❗ Type A dissection involving the ascending aorta is considered a 🔪surgical emergency, with mortality rates of 1%-2% per hour following symptom onset; rapid diagnosis and treatment are critical. Most TAAs (60%) involve the ascending aorta (between the aortic valve and the brachiocephalic artery)

Type A (ascending aorta) dissections can lead to aortic rupture into the pericardial space and hemopericardium, which can rapidly progress to cardiac tamponade and cardiogenic shock.

Type B dissections occur in the proximal descending aorta (distal to the left subclavian artery), and, if stable, may be managed medically by controlling the blood pressure and heart rate to prevent extension of the dissection.

Px: Patients often have diastolic murmurs due to aortic insufficiency from a proximal dissection into the valve. A tracheal tug is considered positive if the pulsating aorta is felt when the trachea is pulled upward, a sign that the expanding aorta is contacting the left mainstem bronchus.

Tx:

The goals of initial therapy of aortic dissection include:

Adequate pain control

Reduction of systolic blood pressure (SBP) to 100-120 mm Hg.

🎺Intravenous beta blockers (eg, labetalol, propranolol, esmolol) are preferred for initial therapy to reduce heart rate, SBP, and LV contractility. These effects lead to a decrease in the rate of rise in SBP (dP/dt) and in aortic wall stress. [Decrease in left ventricular (LV) contractility to reduce aortic wall stress.]

🧨Nitroprusside 🥈 is commonly used to titrate systolic blood pressure to less than 120. Cx: Can cause reflex sympathetic stimulation with consequent rises in heart rate, LV contractility, and aortic wall stress. Used as a second-line agent only if SBP remains above goal (ie, >120 mm Hg) despite adequate beta blockade.

Dx:

  • CT angiography is the initial study of choice in hemodynamically stable patients with no evidence of renal dysfunction. It can reveal an intimal flap separating the true and false lumens in the aorta.
  • MR angiography is more time consuming and requires the administration of gadolinium-containing contrast agents for contrast enhancement; it should be avoided in patients with moderate to severe kidney disease due to the risk of nephrogenic systemic fibrosis.
  • Transesophageal (not transthoracic) echocardiography (TEE) has excellent sensitivity and specificity and is the preferred diagnostic study in patients with ❗ hemodynamic instability or renal insufficiency; a transthoracic echocardiogram may not visualize parts of the aorta well.

Cx: Retrograde extension of the intimal tear can involve the aortic valve and cause acute aortic regurgitation (AR). As seen in this case, affected patients can develop sudden onset of worsening chest pain, hypotension, and pulmonary edema, along with the early decrescendo diastolic murmur of AR.

Ascending aortic dissection can propagate proximally into the pericardial space and lead to hemopericardium and cardiac tamponade.

How well did you know this?
1
Not at all
2
3
4
5
Perfectly
9
Q

Marfan syndrome

A

Skeletal

  • Arachnodactyly
  • ↓ Upper-to-lower body segment ratio, ↑ arm-to-height ratio
  • Pectus deformity, scoliosis, or kyphosis
  • Joint hypermobility

Ocular

  • Ectopia lentis

Cardiovascular

  • Aortic dilation, regurgitation, or dissection
  • Mitral valve prolapse

Pulmonary

  • Spontaneous pneumothorax from apical blebs

Skin

  • Recurrent or incisional hernia
  • Skin striae

Marfan syndrome involves mutations that affect the extracellular matrix protein fibrillin-1 and result in disruption of connective tissue structural integrity throughout the body. The effects of the disease on the aortic root are especially prominent and account for the majority of morbidity and mortality in these patients; dissection, if it occurs, usually occurs prior to age 40.

Aneurysmal aortic root dilation is extremely common in Marfan syndrome (up to 80% of cases) and can frequently lead to chronic aortic regurgitation, identified by an early decrescendo 💎diastolic murmur best heard at the right upper sternal border. Left untreated, the aneurysmal dilation can progress to a type A aortic dissection that can extend to involve the aortic valve annulus, further impairing aortic valve closure and resulting in acute aortic regurgitation. The mortality rate for type A aortic dissection is high, and treatment requires emergent surgical repair.

How well did you know this?
1
Not at all
2
3
4
5
Perfectly
10
Q

AAA

A

The incidence of AAA is higher in men than in women and in whites versus blacks, and it increases with age.

The 5-year risk of rupture is 1% to 2% if the aneurysm is less than 5 cm, but 20% to 40% if the size is greater than 5 cm. Operative repair is typically recommended in asymptomatic individuals when the AAA diameter is greater than 5.5 cm; other indications for surgery are rapid expansion or onset of symptoms.

Hx: Typically causes few symptoms until it markedly expands or ruptures. Pain is the most common initial manifestation and can vary according to aneurysm location. Proximal aneurysms tend to present with upper abdominal, flank, or back pain, whereas distal lesions present with lower abdominal or groin pain. Clasically, substernal chest pain with radiation to the back or midscapular region; often described as “tearing” or “ripping” pain.

Locally contained AAA: Severe abdominal or back pain with syncope, followed by vague discomfort is typical for a ruptured abdominal aortic aneurysm (AAA) that has been locally contained, preventing immediate death. Contained rupture of AAA, when misdiagnosed, is most often mistaken for renal colic, acute myocardial infarction, or diverticulitis.

❗Rupture: The sentinel event of sudden, severe back pain associated with loss of consciousness. Symptoms after that time are likely caused by either local irritation and inflammation related to the rupture and hemorrhage or expansion of the aneurysm against adjacent structures. Leukocytosis and anemia are common. In the event of rupture, hemorrhage usually occurs into the retroperitoneum; because the expanding hematoma may be temporarily contained within the retroperitoneum, patients may remain hemodynamically stable and have a delayed presentation.

Pulse or blood pressure differential useful but uncommonly present.

Dx: Chest radiograph may show a widened mediastinal silhouette, pleural effusion, or both.

In symptomatic patients who are hemodynamically stable, the diagnosis should be confirmed with abdominal CT. Hemodynamically unstable patients require emergency surgical repair with confirmation obtained by rapid bedside ultrasound if necessary.

How well did you know this?
1
Not at all
2
3
4
5
Perfectly
11
Q

TAA (Thoracic aortic aneurysm)

A

Most TAAs (60%) involve the ascending aorta (between the aortic valve and the brachiocephalic artery), and a minority involve the descending aorta (distal to the left subclavian artery).

Patients with TAA are usually asymptomatic until the discovery is made incidentally on chest x-ray, CT scan, or ECG. However, some patients develop chest or abdominal discomfort as the TAA grows to compress surrounding structures. The natural history of TAA is one of slow expansion with a progressive increase in the risk of aortic dissection at larger aortic sizes. Expansion rates for TAA are generally less than those of AAA.

  • Share many of the same risk factors as abdominal aortic aneurysm (AAA). Approximately 25 percent of patients with TAA will also be found to have an AAA
How well did you know this?
1
Not at all
2
3
4
5
Perfectly
12
Q

Esophageal rupture

A

Intense retrosternal pain after vomiting; often associated with ethanol use. Pneumomediastinum on CXR can be seen.

How well did you know this?
1
Not at all
2
3
4
5
Perfectly
13
Q

Esophagitis

A

Burning-type chest discomfort usually precipitated by meals and not related to exertion. It is often worse upon lying down and improved with sitting.

How well did you know this?
1
Not at all
2
3
4
5
Perfectly
14
Q

Musculoskeletal pain

A

Typically more reproducible chest pain. Includes muscle strain, costochondritis, and fracture. Should be a diagnosis of exclusion.

How well did you know this?
1
Not at all
2
3
4
5
Perfectly
15
Q

Cocaine

A

Clinical features

  • Sympathetic hyperactivity - tachycardia, hypertension, dilated pupils
  • Chest pain due to coronary vasoconstriction
  • Psychomotor agitation, seizures

Complications

  • Acute myocardial ischemia
  • Aortic dissection
  • Intracranial hemorrhage

Management of chest pain

  • Benzodiazepines for blood pressure & anxiety
  • Aspirin
  • Nitroglycerin & 🍦 calcium channel blockers for pain
  • Beta blockers contraindicated ❌
  • Fibrinolytics not preferred due to increased risk of intracranial hemorrhage
  • Immediate cardiac catheterization with reperfusion when indicated

Hx: Psychomotor agitation, dilated pupils, atrophic nasal mucosa, hypertension, and acute myocardial ischemia (chest pain, electrocardiogram changes)

Cocaine potentiates sympathomimetic actions by causing inhibition of norepinephrine reuptake into the sympathetic neuron. This causes stimulation of alpha and beta adrenergic receptors and can result in coronary vasoconstriction and increase in heart rate, systemic blood pressure, and myocardial oxygen demand. It also enhances thrombus formation by promoting platelet activation and aggregation.

It can also induce spasm of the coronary circulation even if there is no preexisting coronary artery stenosis.

Tx:

🥞 All patients with acute cocaine toxicity and myocardial ischemia should be treated initially with supplemental oxygen and intravenous benzodiazepines. By reducing sympathetic outflow, benzodiazepines reduce anxiety and agitation, improve blood pressure and heart rate, and alleviate cardiovascular symptoms.

Aspirin retards thrombus formation

Nitrates and calcium channel blockers, being vasodilators, are beneficial for the cocaine-induced coronary artery vasoconstriction.

❌ Beta blockers, including cardioselective agents (eg, metoprolol, atenolol, bisoprolol), should NOT be used in patients with cocaine-induced myocardial ischemia or infarction. Their use can cause unopposed alpha adrenergic stimulation and worsen coronary vasoconstriction

How well did you know this?
1
Not at all
2
3
4
5
Perfectly
16
Q

Panic Disorder

A

May be indistinguishable from angina. Often diagnosed after a negative evaluation for ischemic heart disease. Often associated with palpitations, sweating, and anxiety.

Sudden panic attacks with acute onset of somatic symptoms that may include chest pain, palpitations, sweating, nausea, dizziness, dyspnea, and numbness.

These symptoms usually last from 5 to 60 minutes. Approximately 50% of patients with panic disorder also have associated agoraphobia, with fears of being in crowds or in places from which escape would be difficult.

Normal cardiac and pulmonary examinations

Tx: Cognitive behavioral therapy (CBT) has been shown to be the most effective psychotherapeutic intervention in controlled trials. Selective serotonin reuptake inhibitors and serotonin-norepinephrine reuptake inhibitors have been shown to be effective.

How well did you know this?
1
Not at all
2
3
4
5
Perfectly
17
Q

Anginal Equivalent

A

Occurs when a patient has no chest pain, but has other symptoms of cardiac ischemia (eg,
dyspnea) that is predictably precipitated by exertion and relieved by rest.

How well did you know this?
1
Not at all
2
3
4
5
Perfectly
18
Q

Non Anginal Pain

A

“Stabbing,” “shooting,” “knifelike,” “jabbing,” and “tingling.”.

How well did you know this?
1
Not at all
2
3
4
5
Perfectly
19
Q

GERD

A

Randomized controlled trials have shown that a therapeutic trial of twice-daily PPI treatment is effective in 50% to 60% of patients with noncardiac chest pain

How well did you know this?
1
Not at all
2
3
4
5
Perfectly
20
Q

Differential Diagnosis Dyspnea: Cardiovascular Causes​

A

Aortic stenosis

Mitral stenosis

Mitral regurgitation

Chronic constrictive pericarditis

Cor Pulmonale

Hypertrophic Cardiomyopathy

How well did you know this?
1
Not at all
2
3
4
5
Perfectly
21
Q

Benign Murmurs

Pathologic Murmurs

A

Intensity or grade (<3/6), timing (early and brief systolic), lack of radiation, and absence of additional abnormal heart sounds.

❗ Diastolic and continuous murmurs are usually due to an underlying pathologic cause. Their presence should prompt further evaluation with a transthoracic echocardiogram, which can identify valvular regurgitation and evaluate for any associated structural abnormalities or hemodynamic consequences.

Benign characteristics of a murmur include its intensity or grade (<3/6), timing (early and brief systolic), lack of radiation, and absence of additional abnormal heart sounds.

A midsystolic murmur can be detected occasionally in young, asymptomatic adults. This murmur is usually benign and, in the absence of symptoms or other abnormal findings, does NOT require further evaluation.

Grading the Intensity:

1 - Murmur heard with the stethoscope, but not at first (S > murmur)

2 - Faint murmur heard with the stethoscope on the chest wall (S = murmur)

3 - Murmur heard with the stethoscope on the chest wall; louder than grade 2 but without a thrill (a vibration felt on palpation over the heart) (S

4 - Murmur associated with a (palpablre) thrill

5 - Murmur heard with just the rim of the stethoscope held against the chest

6 - Murmur heard with the stethoscope held close to but not touching the chest wall

Bell of the stethoscope detects low-frequency sounds and murmurs

How well did you know this?
1
Not at all
2
3
4
5
Perfectly
22
Q

Aortic stenosis

A

History of heart murmur, chest pain, syncope, exertional dyspnea; history of rheumatic fever; history of aortic coarctation.

Hx: The three most common causes of aortic stenosis in the general population are senile calcific aortic stenosis, bicuspid aortic valve, and rheumatic heart disease.

Px: 👧🏽Mid- to late-peaking (Crescendo-decrescendo) ejection (mid) systolic murmur at right upper sternal border cardiac base with radiation to ❗❗carotid arteries, an S4 (left atrial kick against a stiff left ventricle)[the high resistance generated by the stenosed aortic valve causes concentric hypertrophy and stiffening of the left ventricle, resulting in the S4]. a single S2 as a result of loss of the aortic closure component, and delayed timing and decreased amplitude in the carotid pulses (pulsus parvus et tardus),

🔪 Surgical aortic valve replacement (AVR) should be considered in patients with severe AS and ≥1 of the following criteria:

  • Presence of symptoms attributable to AS: Patients with symptomatic, severe AS have a relatively high risk of sudden cardiac death.
  • Left ventricular ejection fraction (LVEF) <50%, regardless of symptoms: A depressed LVEF is often due to excessive afterload created by the stenotic valve, and it frequently normalizes with AVR.
  • Undergoing other cardiac surgery (eg, coronary artery bypass grafting): The valve can be repaired concomitantly.

The definition of severe AS encompasses many patients who are asymptomatic because it was designed to identify nearly all patients who may benefit from AVR (high sensitivity). Some of these patients are truly asymptomatic. Others lack symptoms only because of a sedentary lifestyle; when subjected to exertion (eg, stress testing) they have typical severe AS symptoms.

Coarctation of aorta usually presents with a midsystolic murmur over the left interscapular space which may become continuous if the lesion in the vessel is narrowed enough to cause high-velocity jet flow. Classic to this condition are arterial hypertension in the upper extremities and normal or low blood pressure, with diminished or delayed pulsations in the lower extremities. Chest x-ray findings such as sign of 3️⃣ due to indentation of the aorta at the site of coarctation with pre and post-stenotic dilatation and rib notching due to rib erosions by dilated collateral vessels are classic findings.

Valvuloplasty:

Bioprosthetic valves are most commonly xenografts (usually porcine); homografts from cadavers and autografts (from the pulmonary position) are less commonly implanted. Thromboembolic complications are common after implantation of a mechanical heart valve. This increased risk of thromboembolic phenomena is not seen 3 months after implantation of a bioprosthetic valve. Thus, in the absence of atrial fibrillation, long-term anticoagulation is not necessary for most patients who receive a bioprosthetic valve. For many patients this confers significant advantage, as it eliminates the risk of hemorrhagic complications related to long-term anticoagulant therapy. T he major disadvantage of bioprosthetic valves is that the rate of structural deterioration is faster and the expected valve life is shorter. Most mechanical valves have an expected life of 20 to 30 years. In contrast, one-third of patients with porcine bioprosthetic valves will require repeat valve replacement in 10 years, and half will need a new valve in 15 years.

Most experts favor a bio-prosthetic valve for women who are contemplating pregnancy. Mechanical valves require long-term anticoagulation with warfarin, which is teratogenic. Women with mechanical valves who are planning pregnancy should switch to an injectable heparin (which is inconvenient and more costly) before conception and continue this during most of pregnancy. Patients with a prosthetic heart valve are at increased risk of infective endocarditis. For prosthetic heart valve patients, prophylactic antibiotics are recommended before and after some high-risk procedures. T hough official recommendations have recently eliminated many procedures for which antibiotic prophylaxis was traditionally recommended, antibiotic prophylaxis is still recommended for patients undergoing dental procedures that manipulate gingival tissue. T his is recommended for all patients with a prosthetic heart valve irrespective of valve type or location. All patients with a prosthetic heart valve need regular follow-up. Many experts recommend yearly echocardiography beginning 5 years after valve implantation.

Cx: Prosthetic valve dysfunction (PVD), which most commonly occurs in the following forms:

  • Paravalvular leak (regurgitation around the valve): more commonly occurs with mechanical (rather than bioprosthetic) valves and results from dehiscence of the valve from the aortic or mitral annulus, often due to annular degeneration or underlying infective endocarditis.
  • Transvalvular regurgitation (regurgitation through the valve): more commonly affects bioprosthetic (rather than mechanical) valves and can result from cusp degeneration or occasionally valvular thrombus that impairs valve closure. Patients are often initially asymptomatic but can develop severe heart failure; those with significant regurgitation generally have a poor prognosis.

PVD can also involve valvular obstruction (stenosis), which typically results from valvular thrombus or cusp malfunction (ie, failed opening) and presents with a characteristic stenotic, rather than a regurgitant, murmur.

The best initial evaluation for PVD is echocardiography, which allows visualization of the valve and surrounding anatomy. Depending on the cause and extent of dysfunction, further studies and possible surgical intervention may be indicated.

How well did you know this?
1
Not at all
2
3
4
5
Perfectly
23
Q

LUSB (P)

A
How well did you know this?
1
Not at all
2
3
4
5
Perfectly
24
Q

Pulmonic stenosis

A

Etiology

  • Congenital (usually isolated defect)
  • Rarely acquired (eg, carcinoid)

Clinical presentation

  • Severe: Right-sided heart failure in childhood
  • Mild: Symptoms (eg, dyspnea) in early adulthood
  • Crescendo-decrescendo murmur (↑ on inspiration)
  • Systolic ejection click & widened split of S2

Diagnosis

  • Echocardiography

Treatment

  • Percutaneous balloon valvulotomy (preferred)
  • Surgical repair in some cases

Commonly occurs as an isolated congenital defect and rarely occurs as an acquired defect (eg, rheumatic fever, carcinoid syndrome). Severe PS is typically diagnosed early in life due to presentation of right-sided heart failure, but patients with relatively mild PS often remain asymptomatic throughout childhood and develop symptoms (eg, dyspnea with exertion) in early adulthood. Cardiac auscultation reveals a pulmonic ejection click (high-pitch sound after S1 best heard during expiration) followed by a crescendo-decrescendo systolic murmur over the left second intercostal space. The murmur intensifies with inspiration. The stenosis also causes the pulmonic valve to close later than usual, resulting in widened splitting of the aortic and pulmonic components of S2; the splitting is further increased during inspiration.

How well did you know this?
1
Not at all
2
3
4
5
Perfectly
25
Q

LSB

A
How well did you know this?
1
Not at all
2
3
4
5
Perfectly
26
Q

Aortic regurgitation (AR) [Insufficency]

A

Common etiologies

  • Congenital bicuspid aortic valve
  • Postinflammatory (eg, rheumatic heart disease, endocarditis)
  • Aortic root dilation (eg, Marfan syndrome, syphilis)

Pathophysiology

  • Backflow from aorta into LV → ↑ LV end-diastolic volume
  • LV initially compensates with eccentric hypertrophy → ↑ SV & CO
  • Eventual LV dysfunction → ↓ SV & CO → heart failure

Clinical findings

  • Diastolic decrescendo murmur
  • Widened pulse pressure (↑SBP & ↓DBP)
  • Rapid rise-rapid fall (“water-hammer”) pulsation
  • Abrupt carotid distension & collapse, “pistol-shot” femoral pulses

Etiologies may include dissecting aorta, Marfan syndrome, bicuspid aortic valve, rheumatic heart disease, ankylosing spondylitis, endocarditis, and syphilis.

In young patients, Bicuspid aortic valve (BAV) can cause isolated AR due to valvular leaflet abnormalities or aortic root dilation; in older patients, it usually leads to aortic stenosis. Dilation of the aortic root or the ascending aorta can progress to aortic aneurysm and dissection, sometimes causing sudden death. Although some BAV cases develop sporadically, others have a Mendelian inheritance pattern (eg, autosomal dominant with incomplete penetrance).

Px: AR leads to an early decrescendo 💎diastolic murmur [blowing](begins immediately after A2), best heard with the diaphragm of the stethoscope along the left sternal border at the third and fourth intercostal spaces while the patient is sitting up, leaning forward, and holding a breath in full expiration. (all due to the large stroke volume with fast runoff) include a wide arterial pulse pressure and a diastolic rumble (from the aortic regurgitant flow displacing the mitral valve, often called the Austin Flint murmur), Musset sign (head bobbing with the heartbeat), water-hammer pulse or Corrigan pulse (rapidly rising and collapsing pulse), Hill sign (an increase of > 40 mm Hg in femoral artery systolic BP compared to brachial artery BP), Quincke pulse (nail-bed capillary pulsations), pistol-shot pulse (booming sound heard over the femoral arteries), and Duroziez sign (bruit auscultated over the femoral artery when compressed). Carotid arteries have a rapid, accentuated upstroke, with a rapid decline (frequently referred to as a Corrigan pulse); the point of maximal impulse is displaced (suggesting left ventricular volume overload); and the pulse pressure is widened (systolic pressure minus diastolic pressure; normal is ≤40 mm Hg).

Dx: The murmur of aortic regurgitation is a soft, blowing diastolic murmur that is often heard best at the third left or second right intercostal space. It does not radiate well and may be confined to a very limited area of the chest wall. The murmur can be heard best with the patient leaning forward in end-expiration.

The left ventricle initially adapts via the Frank-Starling mechanism, with left ventricular (LV) stretch allowing for an increase in stroke volume to help maintain cardiac output. As the volume overload becomes chronic, the left ventricle further adapts via eccentric hypertrophy, which involves the addition of myocardial fibers in series. This allows for both increased LV complianceto accommodate additional LV volume and increased LV contractility to sustain increased stroke volume and maintain cardiac output.

In the short term, eccentric hypertrophy is beneficial because it allows for an initial asymptomatic periodin the setting of severe chronic AR. However, in the long term, eccentric hypertrophy is maladaptive because it causes increased LV wall stress, which eventually leads to LV contractile dysfunction and decompensated heart failure.

How well did you know this?
1
Not at all
2
3
4
5
Perfectly
27
Q

[HOCM] Hypertrophic Obstructive Cardiomyopathy

A

HCM is an autosomal dominant genetic disorder caused by mutations in one of several sarcomere genes encoding the myocardial contractile proteins of the heart. The 2 most common mutations (responsible for about 70% of identifiable mutations) occur in the cardiac myosin binding protein C gene and the cardiac beta-myosin heavy chain gene. First-degree relatives of an affected patient with a known disease-causing mutation can be offered genetic testing to identify the risk for developing HCM.

Hypertrophic cardiomyopathy (HCM) is typically asymptomatic in childhood and adolescence.

Hx: The clinical manifestations of HCM, which depend on the extent of hypertrophy and outflow tract obstruction, include:

Exertional dyspnea, chest pain, fatigue, palpitations, presyncope, or syncope

Dx: Some patients have systolic anterior motion of the mitral valve, leading to anterior motion of mitral valve leaflets toward the interventricular septum. Contact between the mitral valve and the thickened septum during systole leads to left ventricular outflow tract obstruction (LVOT) and is responsible for the Harsh crescendo-decrescendo systolic murmur heard best at the apex and lower left sternal border. Also associated with an S4

HCM is often characterized by left ventricular hypertrophy most prominent in the basal anterior septum.

Px:

Maneuvers that decrease preload (Valsalva maneuver, standing, nitroglycerin)[reduce left ventricular end diastolic volume and increase the turbulence of blood flow exiting the ventricle during systole] 📢 enhance the murmur, and

Manuvers that 💙increase venous return (leg elevation) or squatting, hand grip [elevation of arterial pressure] 🔇 diminish the murmur.

The typical murmur of hypertrophic cardiomyopathy is a harsh systolic diamond-shaped murmur heard best at the lower sternal border and apex.

Tx: Management includes avoidance of strenuous exertion, good hydration. First-degree relatives should be screened with echocardiography. Patients with symptoms (eg, syncope, heart failure, angina) should be treated with negative inotropic agents (eg, beta blockers, verapamil, disopyramide) as the initial medical therapy.

🎺Beta blockers (eg, metoprolol, atenolol) are the most commonly used agents for initial monotherapy.

ICD implantation is effective for primary prevention of sudden cardiac death in patients with HCM.

Most low-level recreational activities, such as bowling or golf, are probably permissible. High-level recreational activities, such as basketball or bodybuilding, are not advised or are strongly discouraged. Recommendations on moderate-level recreational activities vary. Some activities, such as tennis, are deemed probably permissible, whereas others, including weightlifting, are strongly discouraged. Patients with HCM require formal counseling about acceptable levels of physical activity.

How well did you know this?
1
Not at all
2
3
4
5
Perfectly
28
Q

T

A

VSD

ASD

How well did you know this?
1
Not at all
2
3
4
5
Perfectly
29
Q

Tricuspid regurgitation

A

Tricuspid regurgitation most often is caused by left-sided heart disease that causes pulmonary hypertension, which leads to right ventricular enlargement and annular dilation.

Primary pulmonary hypertension and elevated pulmonary pressure as a result of chronic lung disease also cause tricuspid regurgitation, with the term cor pulmonale describing right-sided heart failure as a result of pulmonary hypertension in the absence of left-sided heart disease. Other causes of tricuspid regurgitation include 🔥 endocarditis, injury after pacer lead placement, carcinoid disease, mediastinal irradiation, and trauma. Carcinoid disease causes direct toxicity to the tricuspid valve, seen as leaflet thickening and retraction.

Px: Characterized by a 💨blowing (holo) systolic murmur at the lower left sternal border that may increase in intensity with inspiration. The murmur does not radiate well, although it can sometimes be heard at the upper left sternal border. A finding characteristic of right-sided murmurs is augmentation in intensity with inspiration.

May be intensified during inspiration and reduced during expiration or with the Valsalva maneuver (Carvallo’s sign). Inspiration increases right heart volume and therefore augments right-sided murmurs.

This murmur is sometimes associated with a prominent right ventricular pulsation along the left parasternal region or regurgitant waves seen in the neck veins.

In mild or moderate tricuspid regurgitation, most patients are asymptomatic.

❗Severe tricuspid regurgitation may be associated with signs of advanced right-sided heart failure, including jugular venous distention (JVD), ascites, hepatomegaly (sometimes pulsatile), and lower extremity edema. Electrocardiography may show enlargement of the right atrium or the right ventricle.

How well did you know this?
1
Not at all
2
3
4
5
Perfectly
30
Q

M

A
How well did you know this?
1
Not at all
2
3
4
5
Perfectly
31
Q

Mitral stenosis (MS)

A

History of rheumatic fever, heart murmur

Hx: In a young patient from a developing country, the clinical presentation of progressive dyspnea, nocturnal cough, and hemoptysis is highly suggestive of rheumatic MS. Long-standing MS leads to an increase in left atrial pressure, which in turn leads to elevated pulmonary pressures and pulmonary vascular congestion; these changes can cause dyspnea, orthopnea, paroxysmal nocturnal dyspnea, and hemoptysis.

Px:💎diastolic murmur with presystolic accentuation (late diastolic accentuation) of the murmur occurs because of increased flow across the mitral valve with atrial contraction. A loud first heart sound and early diastolic opening snap may also be present.

Cx: In addition, the left atrial enlargement resulting from increased left atrial pressure predisposes to the development of atrial fibrillation, PVC’s, SVT, or atrial flutter. AF increases the risk of left atrial thrombus formation and systemic thromboembolic complications (eg, stroke).

How well did you know this?
1
Not at all
2
3
4
5
Perfectly
32
Q

Mitral regurgitation / MVP 🏀

A

History of heart murmur, mitral valve prolapse, or myocardial infarction

Holosystolic murmur at cardiac base (apex), and radiates laterally or posteriorly.

The systolic murmur of chronic MR NOT due to MVP is 📢 intensified by isometric exercise (handgrip) but is REDUCED with the Valsalva maneuver.

Pulmonary edema: As the resistance to retrograde flow across the leaky mitral valve decreases, a larger proportion of stroke volume flows into the left atrium rather than across the aortic valve.

Primarily caused by retrograde flow across the MV.

To increase antegrade flow (thereby increasing cardiac output and decreasing pulmonary vascular congestion) we should reduce the left ventricular afterload. Lower resistance to flow through the LV outflow tract will increase the proportion of stroke volume that enters systemic circulation.

Tx: Vasodilators such as 🃏 ACE inhibitors and 💦hydralazine are frequently used. ☠ Nitroprusside is another consideration.

🏀 MVP: Mitral valve prolapse occurs in approximately 2% of the general population and is the most common cause of mitral regurgitation.

“Click-murmur” complex. This complex includes a midsystolic click, believed to be caused by sudden tensing of the mitral subvalvular apparatus as the leaflets prolapse into the left atrium, followed by a late systolic murmur.

Valsalva maneuver and standing from a squatting position decrease end-diastolic volume and move the click-murmur complex closer to the S1. [Maneuvers that decrease left ventricular volume which exaggerates the propensity of mitral leaflet prolapse.]

Squatting from a standing position increases venous return (ie, preload), which in turn causes an increase in left ventricular size and volume. This leads to a delay in the valve prolapse, with a later click and shorter murmur. Isometric exercise also diminish the degree of prolapse, and the click-murmur is delayed and decreases in intensity.

Surgical indications for severe chronic mitral valve regurgitation

Primary MR

  • Surgery if LVEF 30%-60% (regardless of symptoms)
  • Consider surgery if successful valve repair is highly likely:
    • Symptomatic & LVEF <30%
    • Asymptomatic & LVEF >60%

Secondary MR

  • Medical management, valve surgery rarely indicated

Primary mitral regurgitation (MR) is defined as that caused by an intrinsic defect of the mitral valve apparatus (eg, leaflets, chordae tendineae) and is differentiated from secondary MR, which results from other cardiac disease (eg, myocardial ischemia, dilated cardiomyopathy).

Chronic severe MR of primary etiology is best treated with surgical repair, and the timing of surgery should precede the development of significant left ventricular (LV) dysfunction. An important consideration is that measured LV ejection fraction (LVEF) overestimates LV function in severe MR because regurgitant flow accounts for a large amount of the stroke volume. Therefore, LVEF 30%-60% is generally an indication for surgical repair or replacement in patients with primary chronic severe MR, regardless of symptoms. Patients with LVEF <30% likely have life-limiting, permanent systolic dysfunction, and surgery should be considered only when there is a high likelihood of successful valve repair (rather than replacement). Preemptive surgery can also be considered in asymptomatic patients with LVEF >60% who are excellent candidates for successful valve repair.

How well did you know this?
1
Not at all
2
3
4
5
Perfectly
33
Q

Ddx Acute Dyspnea: CARDIOVASCULAR

A

Heart failure (acute)

Myocardial infarction

Pericardial tamponade

How well did you know this?
1
Not at all
2
3
4
5
Perfectly
34
Q

Congestive Heart failure (acute)

<strong>I</strong> (asymptomatic) 5%-10% Mortality

<strong>II</strong> (symptomatic; slight limitation of physical activity)[are comfortable at rest, but have fatigue, palpitations, dyspnea, or angina with ordinary activity] 15%-30% Mortality

<strong>IIIa </strong>(symptomatic; marked limitation of physical activity)[less-than-ordinary activity causes symptoms] 15%-30% Mortality

<strong>IV</strong> (inability to perform any physical activity without symptoms) [symptoms at rest and increased symptoms with even minor activity] 50%-60% Mortality

A

Hx: Cardiovascular risk factors, CAD, paroxysmal nocturnal dyspnea

Px:

The third heart sound (S3) is a low-frequency diastolic sound produced by the passive ventricular filling during early diastole; it is best heard over the cardiac apex in the left lateral decubitus position. An abnormal S3 (louder and higher pitch, S3 gallop) is commonly heard in patients with CHF due to left ventricular systolic dysfunction, and it correlates with elevated left atrial and/or ventricular filling pressures and serum BNP levels.

S3 gallop 0.99 (specificity), Jugular venous distention 0.92, Pulmonary crackles 0.78, Hepatojugular reflux 0.96, Ascites 0.97, Edema 0.78.

Dx: EKG, Labs, BNP, CRX

BNP is a natriuretic hormone released from ventricular myocytes in response to high ventricular filling pressures and wall stress in patients with CHF. It is derived from the cleavage of the prohormone proBNP, which produces a biologically active BNP and an inert N-terminal proBNP (NT-proBNP). Elevated levels of circulating BNP or NT-proBNP correlate with the severity of left ventricular systolic dysfunction. Conversely, normal values have a very high negative predictive value for CHF as a cause of dyspnea and should prompt a search for noncardiac causes of dyspnea.

BNP >100 pg/mL is highly (90%) sensitive and 73% specific in the diagnosis of patients with heart failure in the emergency department. Patients with chronic heart failure may have BNP levels <100 pg/mL. Studies indicate that a value less than 80 pg/mL has a high (99%) negative predicative value and helps rule out CHF. Most patients with dyspnea due to CHF had plasma BNP levels >400 pg/mL whereas levels <100 pg/mL

Other conditions that raise BNP levels include acute myocardial infarction, pulmonary embolism, chronic kidney disease, older age, and female sex.

BNP is reduced by oBesity. Do not routinely measure BNP in patients with typical signs and symptoms of heart failure.

Chest radiography may be helpful in determining the cause of dyspnea. Once heart failure is diagnosed, serial chest radiographs are not sensitive to small changes in pulmonary vascular congestion and are not recommended.

Echocardiography can help identify specific causes of heart failure. Echocardiography is necessary for distinguishing systolic heart failure from heart failure with preserved systolic function.

Coronary angiography for patients with new-onset heart failure who have angina are potential candidates for revascularization and evaluation for ischemia.

Poor prognositc factors:

Hyponatremia in patients with CHF usually parallels the severity of heart failure and is an independent predictor of adverse clinical outcomes.

JVP, Mitral regurgitation, LBBB,

Tx: A 5-mg dose of oral morphine given four times daily has been shown to help relieve dyspnea in patients with end-stage heart failure.

Rx: Initial therapy:

🃏 ACE inhibitor (enalapril) [EF < 40%] “CONSENSUS TRIAL” (if ACE inhibitor is not tolerated because of cough, an ARB can be used; if ACE inhibitor is contraindicated because of hyperkalemia or renal insufficiency, combined hydralazine and isosorbide dinitrate can be used)

🎺 β-Blocker (carvedilol), ER metoprolol (succinate), and bisoprolol [EF < 40%] “MERIT HF” are approved for the treatment of heart failure. interfere with the harmful effects of sustained activation of the adrenergic nervous system (α1, β1, and β2) by competitively blocking their receptors.

❗ β-blockers should NOT be initiated when a patient is acutely decompensated (hypotensive or volume overloaded w/ pulmonary edema), as initiation of therapy is associated with a transient decline in cardiac output. β-Blockers can be initiated and tolerated once euvolemia or near-euvolemia has been established. [Patients with systolic heart failure should be treated with a β-blocker, regardless of symptom status, including heart failure that is asymptomatic or mildly symptomatic.].

Additional therapy:

🎢 Diuretic as needed to maintain euvolemia [loop diuretics (eg, furosemide, bumetanide, torsemide)] Hx: Dyspnea, orthopnea, paroxysmal nocturnal dyspnea [PND], bibasilar crackles, hypoxemia) are consistent with acute pulmonary edema. ❗ Acute Decompensated Heart Failure (ADHF): Supplemental oxygen and IV loop diuretics (eg, furosemide).

Aldosterone antagonists (spironolactone or 🍎eplerenone) [EF <30%] “RALES TRIAL” [indicated for patients with severe systolic heart failure (NYHA class III-IV symptoms), serum potassium level <5 meq/L, and creatinine level <2.5 mg/dL][should include close clinical and laboratory follow-up, with particular attention to serum potassium levels.]

💣Hydralazine and 🧨isosorbide dinitrate (for black patients) “AHEFT TRIAL”‘

Digoxin is used primarily for symptom control.

A newer-generation dihydropyridine calcium channel blocker (amlodipine) may improve control of blood pressure in a patient with resistant hypertension (blood pressure that is not at the target value with three-drug therapy with different classes of drugs, including a diuretic). Older-generation calcium channel blockers, such as diltiazem, nifedipine, and verapamil, may precipitate EXACERBATION of heart failure because of their negative inotropic effects.

Tx: Limiting dietary sodium to 2 g daily and fluid to 2 liters per day and recording daily weights results in fewer hospitalizations for patients with decompensated heart failure.

Device threapy Indications:

💫Cardiac Resynchronization Therapy: Implantable Cardioverter-Defibrillator (ICD) is an internal defibrillator that senses dangerous cardiac arrhythmias and automatically converts the rhythm to sinus rhythm by either administering a high-energy shock or delivering a short series of paced beats.

Ischemic or nonischemic cardiomyopathy with ejection fraction ≤3️⃣5️⃣% (primary prevention) “DEFINITE TRIAL”

Cardiac Resynchronization Therapy: Biventricular Pacing (pacing of both the right and left ventricles that improves pump function cause by dyssynchrony because of the conduction delay)

ALL of the following:

NYHA Class III or IV

Ejection fraction ≤35%

Ventricular dyssynchrony (QRS duration >120 msec)

Cardiac transplantation improves survival, functional status, and quality of life in patients with NYHA Class III or IV heart failure. Relative contraindications to cardiac transplantation include age >65 years, end-organ damage from diabetes or vascular disease, malignancy, previous stroke, lack of psychosocial support, or active psychiatric illness.

Fu: Monitor your weight, SOB etc.

“Best practice measure” must be seen within 7 days of discharge

How well did you know this?
1
Not at all
2
3
4
5
Perfectly
35
Q

Cor pulmonale

A

Cor pulmonale refers to impaired function of the right ventricle caused by pulmonary hypertension that occurs due to underlying diseases of the lungs (COPD, interstitial lung disease), pulmonary vasculature (idiopathic pulmonary arterial hypertension), or obstructive sleep apnea.

Diseases leading to hypoxic vasoconstriction, as in cystic fibrosis; occlusion of the pulmonary vasculature, as in pulmonary thromboembolism; other pulmonary vascular problems, such as collagen-vascular disease; parenchymal destruction as in sarcoidosis; and COPD; Chest wall disorders (eg, kyphoscoliosis). COPD is the most common cause of cor pulmonale in the United States, with nearly 25% of COPD patients developing this disorder

By convention, right ventricular dysfunction due to left heart disease or congenital heart disease is NOT considered cor pulmonale.

Physical examination may show loud P2 (pulmonic component of the 2nd heart sound), tricuspid regurgitation murmur (holosystolic at the left lower sternal border), elevated jugular venous pressure (JVP), peripheral edema, hepatomegaly due to hepatic congestion, and possible ascites. COPD patients usually have distant heart sounds due to hyperinflated lungs. End-stage cor pulmonale may present with hypotension, tachycardia, and other signs of cardiogenic shock due to decreased stroke volume.

Chest x-ray may show enlarged central pulmonary arteries and loss of retrosternal air space due to right ventricular hypertrophy.

📈 The electrocardiographic findings include tall peaked P waves in leads II, III, and aVF (indicating right atrial enlargement), tall R waves in leads V1 to V3 and a deep S wave in V6 with associated ST-T wave changes (indicating right ventricular hypertrophy) and right axis deviation. Right bundle branch block occurs in 15% of patients.

Right heart catheterization is the 🥇gold standard for diagnosis and typically shows elevated central venous pressure, right ventricular end-diastolic pressure, and mean pulmonary artery systollic pressure (>25 mm Hg at rest) without left heart disease.

Tx: Involves optimizing right ventricular dynamics (preload, afterload, and contractility) with supplemental oxygen, diuretics, treatment of underlying etiology, and intravenous inotropes for severe decompensation.

Dobutamine is an adrenergic agonist with predominant activity on beta-1 receptors and minimal activity on beta-2 and alpha-1 receptors. It is used for the management of severe heart failure associated with severe left ventricular systolic dysfunction and cardiogenic shock. Stimulation of beta-1 receptors results in increased production of cAMP in cardiac myocytes, which in turn leads to enhanced calcium-mediated binding of the actin-myosin complex to troponin C and increased myocardial contractility (positive inotropic effect). Heart rate is also increased via calcium channel activation (positive chronotropic effect). The increase in myocardial contractility allows for forward ejection of a higher volume of blood and results in a decrease in left ventricular end-systolic volume. Cardiac output is increased to perfuse organs and less blood backs up into the pulmonary circulation, resulting in improvement of symptoms.

How well did you know this?
1
Not at all
2
3
4
5
Perfectly
36
Q

HFpEF

(diastollic dysfunction)

A

Diastolic dysfunction is caused by impaired myocardial relaxation or increased LV wall stiffness (decreased compliance), leading to increased LV end-diastolic pressure (LVEDP). The increase in LVEDP is transmitted to the left atrium and pulmonary veins and capillaries, causing pulmonary congestion, dyspnea, and exercise intolerance.

Hx: Exertional dyspnea, orthopnea (choking sensation/dyspnea when lying flat), bibasilar rales, lower extremity edema,

Dx: Normal ejection fraction on echocardiography

There is NOT ❓ a large body of evidence to guide the treatment of HFPEF.

Tx: Primarily focused on managing the manifestations of heart failure (volume overload) and targeting risk factors for left ventricular hypertrophy (primarily hypertension), which is strongly associated with HFPEF.

Rx: The angiotensin receptor blocker (ARB) candesartan is an agent that has been studied in a large randomized controlled trial of HFPEF treatment and was associated with a reduction in hospitalizations. Other appropriate agents for use in HFPEF include angiotensin-converting enzyme inhibitors, nondihydropyridine calcium channel blockers (verapamil, diltiazem), and β-blockers.

How well did you know this?
1
Not at all
2
3
4
5
Perfectly
37
Q

Dilated Cardiomyopathy

A

A diagnosis of dilated cardiomyopathy requires evidence of dilatation and impaired contraction of the left ventricle or both ventricles.

Dilated cardiomyopathy has many causes, with the most common being idiopathic (50%), myocarditis (9%), ischemic (7%), peripartum (4%), and toxic (3%).

Idiopathic cardiomyopathy is diagnosed if there is no evidence of coronary artery obstruction, myocarditis, or a primary or secondary form of heart muscle disease.

🍺 Alcoholic cardiomyopathy is a diagnosis of exclusion in patients with dilated cardiomyopathy and history of alcohol abuse in whom no other potential causes of cardiomyopathy (eg, coronary artery disease, valvular heart disease) are suspected or identified. The degree of LV dysfunction in alcoholic cardiomyopathy is directly related to the daily amount and overall duration of alcohol intake. The primary therapy for such patients is complete abstinence from alcohol use; this intervention is associated with improvement or normalization of LV function over time.

Viral or idiopathic myocarditis is most commonly seen following Coxsackievirus B infection, and occurs in about 3.5 - 5% of infected patients. Other viruses commonly implicated include parvovirus B19, human herpesvirus 6, adenovirus, and enterovirus. Viral myocarditis can cause dilated cardiomyopathy via direct viral damage and as a result of humoral or cellular immune responses to persistent viral infections.

Acute myocarditis is immunologically mediated damage to the myocardium; cardiac troponin levels are typically elevated, indicating some degree of myocardial necrosis, and ventricular dysfunction may be global or regional.

Peripartum cardiomyopathy occurs during the last trimester of pregnancy or up to 6 months postpartum in the absence of an identifiable cause. Peripartum cardiomyopathy is a major cause of pregnancy-related death in North America; maternal death is related to heart failure, thromboembolic events, and arrhythmias. Left ventricular function improves within 6 months after delivery in approximately 50% of women with peripartum cardiomyopathy. Subsequent pregnancies are associated with a high risk of recurrence, however. The diagnosis of PPCM can be difficult to make in late pregnancy as many symptoms and signs (eg, dyspnea, edema) are similar to the changes seen in normal pregnancy. A transthoracic echocardiogram showing a dilated left ventricular cavity with global systolic dysfunction and ejection fraction <45% can confirm the diagnosis. Management is similar to that of other forms of heart failure.

How well did you know this?
1
Not at all
2
3
4
5
Perfectly
38
Q

Viral myocarditis

A

Etiology

  • Coxsackievirus B, adenovirus

Clinical features

  • Viral prodrome
  • Heart failure: Respiratory distress, murmur, hepatomegaly

Diagnosis

  • Chest x-ray: Cardiomegaly, pulmonary edema
  • ECG: Sinus tachycardia
  • Echocardiogram: Decreased ejection fraction
  • Biopsy (gold standard): Inflammation, necrosis

Treatment

  • Supportive (eg, diuretics, inotropes)
  • Intravenous immunoglobulin

Myocarditis is a potentially lethal disorder of the myocardium and is most commonly caused, in children, by viral infection (eg, Coxsackievirus B, adenovirus). Myocyte necrosis by direct viral injury and autoimmune inflammation results in impaired systolic and diastolic function.

A viral prodrome (eg, upper respiratory infection [URI]) often precedes the illness. Patients then typically develop chest pain and respiratory distress (eg, dyspnea, tachypnea, wheezing, crackles) from acute left heart failure and pulmonary edema. Dilated cardiomyopathy with mitral regurgitation can cause an S3 gallop and holosystolic murmur. In addition, hepatomegaly is a sign of passive congestion from right heart failure.

Initial workup includes ECG, chest x-ray, and echocardiogram.

CRX typically reveals cardiomegaly (cardiothoracic ratio >50%), and echocardiogram often shows global hypokinesis with decreased ejection fraction. Although endomyocardial biopsy is the diagnostic gold standard, treatment (eg, diuretics, inotropes) is typically initiated based on clinical suspicion.

Cx: In addition, patients should be monitored in the intensive care unit due to risk of shock and fatal arrhythmias.

How well did you know this?
1
Not at all
2
3
4
5
Perfectly
39
Q

Pericarditis

A

Most cases of acute pericarditis are thought to be due to viral infection, and many viruses have been implicated (eg, adenovirus, coxsackievirus, echovirus, influenza virus, HIV).

Etiology

  • Viral or idiopathic
  • Autoimmune disease (eg, SLE)
  • Uremia (acute or chronic renal failure)
  • Postmyocardial infarction
    • Early: Peri-infarction pericarditis
    • Late: Dressler syndrome

Viral, idiopathic, Autoimmune disease (eg, SLE), Uremia (acute or chronic renal failure), Postmyocardial infarction, Early: Peri-infarction pericarditis, Late: Dressler syndrome.

In developing countries and endemic areas (eg, Africa, India & China), tuberculosis is a common cause of constrictive pericarditis. In the United States, the most common causes include idiopathic or viral pericarditis (>40%), radiation therapy (~30%), cardiac surgery (~10%), and connective tissue disorders.

Clinical features & diagnosis

  • Pleuritic chest pain (↓ when sitting up) ± fever
  • Pericardial friction rub (highly specific)
  • ECG: Diffuse ST elevation & PR depression
  • Echocardiogram: Pericardial effusion

Cx:

Uremia Tx: Dialysis is indicated for uremic pericarditis and typically leads to resolution of symptoms.

❗Cardiac tamponade is due to fluid accumulation in the pericardial cavity that increases the intrapericardial pressure above the diastolic ventricular pressure. This restricts venous return to the heart and lowers right and left ventricular filling. The net result is decreased preload, stroke volume, and cardiac output. Inspiration worsens this condition by lowering the intrathoracic pressure and increasing venous return to the right ventricle. Under normal conditions, the right ventricle is able to accommodate this increased venous return by expanding the right ventricular free wall. Hx: Cardiac tamponade may follow trauma or surgery and also may be a complication of malignancy (ie, lung, breast, lymphoma), chronic renal failure, or hypothyroidism. Px: The three classic features of cardiac tamponade, or the Beck triad (hypOtension, soft or absent heart sounds, and JVD) with a prominent x descent but absent y descent. Lung examination typically shows clear lungs to auscultation due to decreased ventricular filling (preload) rather than volume overload. Pulsus paradoxus, an inspiratory drop (from expiration) in systolic blood pressure of more than 10 mm Hg (normal <10 mm Hg). Pulsus paradoxus may also be seen in severe asthma and constrictive pericarditis.

Dx: ECG in tamponade may show low voltage and pulsus alternans:

1) sinus tachycardia; 2) low QRS voltages; and 3) electrical alternans

Electrical alternans in cardiac tamponade is marked by beat-to-beat alternation of the QRS complex (and other waveform components) usually with sinus tachycardia. This so-called 2:1 alternans is due to mechanical effects of the heart swinging to-and-fro in the pericardial effusion. With tamponade, the heart systematically shifts in position on a beat-to-beat basis, accounting for the alternating QRS vector and amplitude in multiple leads.

Tx: Pericardiocentesis should be performed in patients with symptoms or signs suggesting cardiac tamponade.

Chest radiograph may show enlargement of the cardiac shadow (globular-shaped).

How well did you know this?
1
Not at all
2
3
4
5
Perfectly
40
Q

Pericardial Effusion

A

Pericardial effusions are accumulations of fluid between the visceral (epicardium) and parietal pericardium.

Hx: Several factors may lead to pericardial effusions including blockage of the lymphatic or venous systems by tumors, changes in osmostic or oncotic pressures due to metabolic diseases, or increased permeability of the pericardium due to inflammation. Blood in the pericardium (hemopericardium)[trauma] may be an important clue to post operative bleeding. Effusions which do not raise the intrapericardial pressure more than 3 or 4 mmHg will not cause symptoms.

Dx: The best evidence to determine if a pericardial effusion will become hemodynamically significant is to monitor how quickly it is accumulating. Best measured with echocardiography.

CRX: Radiographically, pericardial effusions appear as changes in the size and shape of the cardiac silhouette resulting a featureless, globular or “water bottle” shape.

💀 ICU: The pericardial fluid on an ICU film is generally not distinctly visible; instead it enlarges the cardiac shadow.

Causes an enlarged heart shadow that is often globular shaped (transverse diameter is disproportionately increased). A “fat pad” sign, a soft tissue stripe wider than 2 mm between the epicardial fat and the anterior mediastinal fat can be seen anterior to the heart on a lateral view. Serial films can be helpful in the diagnosis especially if rapid changes in the size of the heart shadow are observed. Approximately 400-500 ml of fluid must be in the pericardium to lead to a detectable change in the size of the heart shadow on PA CXR. Pericardial effusion can be definitively diagnosed with either echocardiography or CT.

Cx: It can be critical to diagnose pericardial effusion because if it is acute it may lead to cardiac tamponade, and poor cardiac filling. In the postoperative patient it could be a sign of bleeding, necessitating a return to the OR.

Dx: Jugular venous distention, clear lungs, pulsus paradoxus, hypotension

History of trauma, preceding “flu” symptoms, collagen vascular disease

How well did you know this?
1
Not at all
2
3
4
5
Perfectly
41
Q

Tamponade

A

Etiology

  • Aortic aneurysm or postmyocardial infarction
  • Malignancy or radiation therapy
  • Infection (eg, viral, tuberculosis)
  • Connective tissue disease (eg, SLE)
  • Cardiovascular surgery

Clinical signs

  • Beck triad: Hypotension, JVD, ↓ heart sounds
  • Pulsus paradoxus (SBP ↓ >10 mm Hg with inspiration)

Diagnosis

  • ECG: Low-voltage QRS, electrical alternans
  • Chest x-ray: Enlarged cardiac silhouette, clear lungs
  • Echocardiogram: Right atrial & ventricular collapse, plethora of the IVC

Subacute tamponade typically develops from chronic processes that cause the slow accumulation of pericardial fluid (eg, malignancy, renal failure), giving the pericardium time to progressively stretch. The pericardium may accommodate 1-2 L of fluid before intrapericardial pressure rises and compromises cardiac function. This large quantity of pericardial fluid causes the classic globular cardiac silhouette on x-ray.

Acute tamponade occurs due to rapid fluid accumulation (eg, accumulation of blood due to blunt cardiac injury). Because the stiff pericardium does not have time to adapt, a small amount of pericardial fluid (eg, 100-200 mL) can dramatically increase intrapericardial pressure without causing cardiomegaly on x-ray (typically seen with >200 mL). Therefore, acute tamponade is associated with normal cardiac contours.

On physical examination, patients may have hypotension with elevated jugular venous pressure (obstructive shock due to the pericardial fluid preventing adequate cardiac contraction) as well as muffled heart sounds (Beck triad, only seen in a minority of patients).

📉 RH Cath: Characteristically, there is elevation and equalization of intracardiac diastolic pressures (right atrial, right ventricular, and pulmonary capillary wedge pressure suggestive of left atrial pressure).

How well did you know this?
1
Not at all
2
3
4
5
Perfectly
42
Q

Constrictive pericarditis

A

Constrictive pericarditis impairs ventricular filling during diastole, causing patients to experience symptoms related to decreased cardiac output (fatigue and dyspnea on exertion) and signs of venous overload (elevated JVP, ascites, and pedal edema). Substernal chest discomfort that can be sharp, dull, or pressure-like in nature, often relieved with sitting forward.

Kussmaul’s sign, defined as lack of the typical inspiratory decline in central venous pressure, and the presence of a pericardial knock (early heart sound after S2) may also be seen. Sharp x and y descents are characteristically seen on central venous tracing. Pericardial calcifications can sometimes be seen on chest x-ray and, when present, help confirm the diagnosis.

Dx:

ECG characteristically shows diffuse PR depression (due to inflammation of atrial myocardium) and diffuse ST elevation (due to inflammation of ventricular myocardium), but findings can vary. Pericardial effusion is present in >50% of cases.

Echocardiographic findings of restrictive filling and ventricular interdependence (ie, diastolic filling of one ventricular chamber that impedes that of the other, as may be manifested by a to-and-fro diastolic motion of the ventricular septum).

Treatment

  • 🧯 NSAIDs & 🌿colchicine for viral or idiopathic
  • Idiopathic or acute viral pericarditis: 🧯Nonsteroidal anti-inflammatory drugs (eg, aspirin, naproxen, ibuprofen, indomethacin) and/or 🌿colchicine are generally used as first-line agents for most patients.
  • 🌑Oral steroids are second line
How well did you know this?
1
Not at all
2
3
4
5
Perfectly
43
Q

MVP

A

Several nonspecific symptoms (atypical chest pain, dyspnea, palpitations, dizziness, anxiety, and panic disorder) and nonspecific electrocardiographic changes have been attributed to MVP. These signs and symptoms in patients with MVP is often called MVP syndrome. However, MVP syndrome is not a validated entity, and patients should be reassured about the benign nature of the symptoms.

Px:

A short systolic murmur at the cardiac apex that disappears with squatting.

Nonejection clicks and/or mid to late systolic murmurs of mitral regurgitation (MR). Squatting from a standing position increases venous return (ie, preload), which in turn causes an increase in left ventricular size and volume. This leads to a delay in the valve prolapse, with a later click and shorter murmur. However, in patients with severe MR (from MVP or otherwise), the murmur may increase in intensity with squatting due to an increase in afterload. Echocardiogram is used to confirm the diagnosis.

How well did you know this?
1
Not at all
2
3
4
5
Perfectly
44
Q

High-output heart failure

A

A-V Fistula’s can be congenital or acquired ( and creates an abnormal connection between the arterial and venous systems that bypasses the capillary beds. Shunting of a large amount of blood through the fistula decreases systemic vascular resistance, increases cardiac preload, and increases cardiac output.

Other causes of high-output cardiac failure include thyrotoxicosis, Paget disease, anemia, and thiamine deficiency.

Dx: Doppler ultrasonography is the preferred test to diagnose an AVF in the extremity, and surgical therapy is indicated for a large AVF.

Hx: Clinical signs include widened pulse pressure, strong peripheral arterial pulsation (e.g., brisk carotid upstroke), systolic flow murmur, tachycardia, and usually flushed extremities. The left ventricle hypertrophies, and the point of maximal impulse is displaced to the left.

Dx: An ECG usually shows left ventricular hypertrophy.

In patients with AVF and significant AV shunting, there is a compensatory increase in the heart rate and stroke volume to meet the oxygen requirements of the peripheral tissues. Normal hearts are usually capable of increasing the stroke volume and cardiac output without significant problems. However, cardiac function can occasionally decompensate over a period of time and result in cardiac failure. Such patients are considered to have heart failure (despite their higher cardiac output) because the circulation is unable to meet the oxygen demand of the peripheral tissues.

How well did you know this?
1
Not at all
2
3
4
5
Perfectly
45
Q

VSD

A

Defect normally causes a loud holosystolic murmur with maximal intensity over the third or fourth left intercostal space.

The intensity of the holosystolic murmur increases with squatting.

How well did you know this?
1
Not at all
2
3
4
5
Perfectly
46
Q

ASD

A

Wide and fixed splitting of the S2. An ejection systolic murmur can be heard over the left second intercostal space due to increased blood flow across the pulmonic valve. Some patients with large ASDs and significant left-to-right shunting also have a mid-diastolic (rumbling) murmur resulting from increased flow across the tricuspid valve.

Paradoxical embolism, a venous thrombus (usually from the leg or pelvic veins) passes into the systemic circulation through an intracardiac defect, typically an atrial septal defect (ASD) or less commonly through a ventricular septal defect (VSD).

How well did you know this?
1
Not at all
2
3
4
5
Perfectly
47
Q

Differential Diagnosis of Syncope:

A
How well did you know this?
1
Not at all
2
3
4
5
Perfectly
48
Q

Neurocardiogenic (Vasovagal) syncope

A

Clinical presentation

  • Inciting event (eg, stress, prolonged standing)
  • Prodrome (eg, pallor, nausea, diaphoresis)
  • Consciousness regained rapidly (eg, <1 minute)

Diagnosis

  • Mainly clinical diagnosis
  • Upright tilt table testing in uncertain cases

Treatment

  • Reassurance
  • Avoidance of triggers
  • Counterpressure techniques for recurrent episodes

Vasovagal Neurocardiogenic syncope (the common “faint”).

Occurs due to an alteration in autonomic drive.

Most commonly, abrupt parasympathetic activation leads to a cardioinhibitory response that manifests as bradycardia with sinus arrest. Less commonly, decreased sympathetic drive leads to a vasodepressor response, resulting in peripheral vasodilation and abrupt hypotension. In both scenarios, cerebral perfusion suddenly decreases and syncope occurs. The episode resolves quickly (typically <1 minute) as cerebral perfusion is rapidly restored.

Vasovagal syncope most commonly affects young patients and is often triggered by emotional stress (eg, fear of venipuncture) or prolonged standing.

🌪🌫🌦🌤⛅Presyncopal symptoms (prodrome), such as lightheadedness, nausea, warmth, diaphoresis, or blurred vision for usually longer than 10 seconds. Common triggers include micturition, defecation, cough, fear, pain, phlebotomy, prolonged standing, postural change, hot environments, emotional distress, dehydration, and use of diuretics or vasodilators.

The specific triggers for reflex syncope cause an alteration in the autonomic response and can lead to a cardioinhibitory, vasodepressor, or mixed response:

  • Increased parasympathetic stimulation can manifest as profound bradycardia, varying degrees of atrioventricular block, or asystole.
  • Decreased sympathetic output can lead to vasodilation, hypotension, or syncope.

+Prodrome

Wasovagal”

visceral organs

Syncope precipitated by pressure on the carotid sinus baroreceptors (tight collar, sudden turning of head). Carotid massage may be confirmatory (Carotid sinus hypersensitivity).

psychiatric

Px: Carotid Massage; decrease in SBP 50 pts; asystole 3 sec

Dx:

12-lead electrocardiography remains the first and most widely recommended test to perform in patients being evaluated for syncope, partly owing to its noninvasive nature, availability, and low cost.

Echocardiography is recommended in patients suspected of having structural heart disease. If an arrhythmia is suspected, documentation of the arrhythmia is indicated either by inpatient telemetry or ambulatory monitoring.

Tilt-table testing is useful in evaluating recurrent syncope in the absence of heart disease to discriminate neurocardiogenic from orthostatic syncope and to evaluate frequent syncope in patients with psychiatric disease. Tilt-table testing may also have a role in evaluating patients in whom documenting neurocardiogenic syncope is important (such as in high-risk occupational settings) and differentiating the cause of syncope from neurologic (such as seizure) or psychiatric etiologies.

Tx: The prognosis is excellent.

For patients with recurrent syncope, management consists of advising patients to avoid triggers and to assume a supine position with leg raising at the onset of symptoms. Physical counterpressure maneuvers (eg, leg crossing with tensing of muscles, handgrip and tensing of arm muscles with clenched fists) during the prodromal phase can improve venous return and cardiac output, sometimes aborting syncopal episodes. Some patients in high-risk settings associated with a potential risk of physical injury (eg, airline pilots, commercial drivers) require temporary activity restriction until these maneuvers are proven to be effective.

How well did you know this?
1
Not at all
2
3
4
5
Perfectly
49
Q

carotid sinus hypersensitivity

A

“w”asovagal

Baroreceptors of the carotid sinus are activated and pass impulses through
the glossopharyngeal nerve to the medulla.

Hx: Syncope occurs in an older patient as a result of head turning, wearing a tight shirt collar, or shaving over the neck area.

Tx: Gentle massage of one carotid sinus at a time may show a period of asystole or hypotension. This should be performed in a controlled setting with monitoring and atropine available.

How well did you know this?
1
Not at all
2
3
4
5
Perfectly
50
Q

Orthostatic hypotension

A

Elderly patients often have impaired baroreceptor sensitivity (autonomic failure), making them particularly susceptible to orthostatic hypotension and orthostatic syncope in the setting of hypovolemia.

Results from insufficient constriction of resistance and capacitance blood vessels in the lower extremities on standing, which may be due to a defect in autonomic reflexes, decreased intravascular volume, or medications. Arterial stiffness, decreased norepinephrine content of sympathetic nerve endings, and reduced sensitivity of the myocardium to sympathetic stimulation all contribute to a tendency toward orthostatic hypotension with age.

Orthostatic syncope is more common in those taking vasoactive drugs, diuretics, or alcohol; and in the setting of volume depletion or autonomic failure or disorders of the autonomic nervous system (idiopathic hypotension, Shy-Drager syndrome), or idiopathic autonomic neuropathy

May be caused by hypovolemia, drugs (α-adrenergic blockers),

Hx: Syncope occurs on assuming the upright position. Patients commonly have symptoms of dizziness, weakness, and fatigue, both before and after the event.

Orthostasis

Volume depletion: D/D/D/H (Diarrhea, dehydration, diuresis, hemorrhage)

ANS dysfunction: DM (autonomic neuropathy)/Parkinsons/Elderly (multiple system atrophy)

Abnormal drop in blood pressure with standing (>20 mm Hg systolic or 10 mm Hg diastolic)

  • SBP decreases 20
  • DBP decreases 10
  • HR increases 10

Dx: Hypovolemia causes decreased renal perfusion, leading to activation of the renin-angiotensin-aldosterone system. Aldosterone stimulates aggressive sodium reabsorptionin the collecting tubules of the kidney in an effort to sustain blood volume. Consequently, most patients with hypovolemia (unless taking diuretics or experiencing significant renal impairment) have decreased urine sodium. The fractional excretion of sodium (ratio of renal sodium clearance to renal creatinine clearance) is typically <1%.

How well did you know this?
1
Not at all
2
3
4
5
Perfectly
51
Q

Obstruction to outflow

Mechanical Cardiac

A

🏃🏽‍♂️ Exertional syncope usually portends an underlying pathologic cause, including ventricular arrhythmias (due to myocardial ischemia/infarction) and outflow tract obstruction (eg, aortic stenosis [AS], hypertrophic cardiomyopathy).

The clinical presentation of progressive dyspnea on exertion, fatigue, and exertional syncope is suggestive of outflow obstruction.

Patients with fixed outflow obstruction cannot increase cardiac output in response to exercise-induced vasodilation, leading to hypotension, transient cerebral hypoperfusion, and decreased exercise tolerance, presyncope, or syncope.

Specific causes include aortic stenosis, hypertrophic cardiomyopathy, mitral stenosis, myxoma, pulmonic stenosis, massive pulmonary embolism, and pulmonary hypertension. (-) Prodrome.

Mitral Stenosis: Px: An opening snap after the S2, followed by a low-frequency decrescendo murmur (diastolic “rumble”). An accentuated P2 (evidence of elevated pulmonary artery pressure), an opening snap (a high-pitched apical sound best heard with the diaphragm), and a low-pitched, rumbling diastolic murmur best heard at the apex using the bell, with the patient in the left lateral decubitus position. Presystolic accentuation of the murmur may be present in both sinus rhythm and atrial fibrillation. As the severity of the stenosis worsens, the opening snap moves closer to S2 as a result of increased left atrial pressure, and the murmur increases in duration.A prominent a wave in the jugular pulse (decreased right ventricular compliance with pulmonary hypertension), a palpable thrill at the apex, a right ventricular heave, and signs of right-sided heart failure (eg, jugular venous distention, hepatomegaly, ascites, edema).

Aortic stenosis: Murmur is diamond-shaped, loudest at the right sternal border, and radiates to the carotid arteries. Delayed (slow-rising) and diminished (weak) carotid pulse (“pulsus parvus and tardus”). Presence of single and soft second heart sound (S2). Mid- to late-peaking systolic murmur with maximal intensity at the second right intercostal space radiating to the carotids

Aortic regurgitation: Blowing diastolic murmur that is best heard at the second right and third left interspaces (left sternal border). Additionally, the carotid arteries have a rapid, accentuated upstroke, with a rapid decline (frequently referred to as a Corrigan pulse); the point of maximal impulse is displaced (suggesting left ventricular volume overload); and the pulse pressure is widened (systolic pressure minus diastolic pressure; normal is ≤40 mm Hg). Tx: Aortic valve replacement surgery is recommended in patients with severe aortic regurgitation and cardiopulmonary symptoms. In asymptomatic patients with severe regurgitation, surgery is recommended when there are signs of adverse hemodynamic effects on the left ventricle, when there is left ventricular enlargement, or when the ejection fraction is less than 55%.

Mitral stenosis :Opening snap followed by a diastolic murmur that is accentuated with atrial contraction. S1 is usually loud; S2 may be variable in intensity. Electrocardiogram in patients with mitral stenosis typically shows features of left atrial enlargement and hypertrophy and right axis deviation.

Mitral regurgitation: Holosystolic murmur that begins shortly after S1 and ends just before S2, heard loudest at the apex, and radiating to the axilla.

Mitral valve prolapse: A “click-murmur” complex. This complex includes a midsystolic click, believed to be caused by sudden tensing of the mitral subvalvular apparatus as the leaflets prolapse into the left atrium, followed by a late systolic murmur. Performing the Valsalva maneuver and standing from a squatting position decrease end-diastolic volume and move the click-murmur complex closer to the S1.

Tricuspid regurgitation: Systolic murmur, loudest at the lower left sternal border, and becomes louder with inspiration.

Benign (innocent) flow murmur: typically midsystolic grade 1 to 2/6 murmurs associated with normal heart sounds and no other findings.

Hypertrophic cardiomyopathy is associated with a harsh crescendo-decrescendo systolic murmur that begins slightly after S1 and is heard best at the apex and lower left sternal border. Performing the Valsalva maneuver and standing from a squatting position increase the intensity of the murmur. The murmur of hypertrophic cardiomyopathy is the only murmur that increases in intensity with the Valsalva maneuver [accentuated during maneuvers that decrease preload (Valsalva maneuver) but is attenuated by increasing afterload (handgrip maneuver)]. Echocardiographic findings confirm asymmetric septal hypertrophy that is consistent with hypertrophic cardiomyopathy. The hypertrophy may be concentric (particularly if marked), but may also disproportionately involve the septal, anterior, lateral, or apical walls. Dynamic left ventricular outflow tract or midcavity obstruction is a feature of hypertrophic cardiomyopathy. Additional echocardiographic features include a small left ventricular cavity and significant left atrial enlargement. Although patients with hypertrophic cardiomyopathy may present with symptoms such as dyspnea, chest pain, or dizziness, many are asymptomatic.

Dx: Transthoracic echocardiography is recommended for diagnosis of systolic murmurs that are grade 3/6 or greater in intensity, diastolic murmurs, continuous murmurs, holosystolic murmurs, late systolic murmurs, murmurs associated with ejection clicks, or murmurs that radiate to the neck or back.

Transesophageal echocardiography may be useful in patients in whom a transthoracic study does not provide adequate diagnostic information or to evaluate the feasibility of surgical repair when surgery is planned. (or cardiac MRI or CT)

Ambulatory electrocardiography may be diagnostically useful in patients with repetitive, frequent palpitations,

How well did you know this?
1
Not at all
2
3
4
5
Perfectly
52
Q

TachyArrhythmia (Fatal syncope)

A

Cardiac diseases predominate as cause of syncope in elderly adults and include bradycardias (sinus and atrioventricular node dysfunction) as well as tachyarrhythmias (supraventricular and ventricular)..

Ventricular arrhythmias causing syncope typically occur in the setting of structural heart disease (myocardial infarction–associated ventricular tachycardia) or with a family history of sudden cardiac death (long QT syndrome, Brugada syndrome). Extended electrocardiographic recording, event monitoring, or electrophysiologic studies may be required to document arrhythmia. ❗ Sudden onset (-) NO Prodrome.

Patients with an arrhythmogenic cause of syncope usually have less than 5 seconds of warning symptoms before each episode.

Dx:

ECG = Holter - Perform in all patients with unexplained syncope. Arrhythmias, conduction defects predisposing to complete heart block, and evidence of structural heart disease may be documented. Yields a diagnosis in approximately 5% of patients in whom the initial history and physical examination are nondiagnostic.

Ambulatory (24-h) ECG monitoring correlates symptoms with an arrhythmia in only 4% of patients. Increasing the duration of monitoring (ie, to 48 or 72 h) increases the number of arrhythmias detected but not the diagnostic yield. Cardiac event monitors (patient triggered or looping) worn for longer periods of time (weeks to months) may be helpful in selected patients with infrequent episodes of suspected arrhythmogenic syncope.

Event Recorder

Implantable loop recorders (ILR) - Device that is placed subcutaneously that has a looping memory capable of storing ECG rhythm events over a period of months to years. Indicated in patients with recurrent, unexplained syncope. Long-term follow-up (median, 17 mo) led to diagnosis of the cause of syncope in 41% of patients compared with 7% of patients assigned to conventional evaluation. An ILR is placed subcutaneously under local anesthesia and has a solid-state looping memory capable of storing ECG rhythm events, with a total capacity of up to 42 minutes. Battery life is approximately 3 years.

Lytes: K+ Mg++

How well did you know this?
1
Not at all
2
3
4
5
Perfectly
53
Q

BradyArrhythmia

A

Hx: Conduction system abnormalities, including prolonged PR interval and intraventricular conduction delay (prolonged QRS-complex duration). Atrioventricular block with associated bradyarrhythmia can be quite intermittent.

May be associated with symptoms of near-syncope (transient) or signs of diminished cardiac output (persistent). Can be diagnosed by electrocardiography, extended electrocardiographic monitoring, or electrophysiologic studies. Includes both sinoatrial and atrioventricular node dysfunction, which may be drug-induced (β-blockers, calcium channel blockers, antiarrhythmic drugs). ❗ Sudden onset (-) NO Prodrome.

How well did you know this?
1
Not at all
2
3
4
5
Perfectly
54
Q

Neurogenic

A

Invariably associated with neurologic signs and symptoms. Carotid Doppler ultrasonography is not indicated because ischemia of the anterior cerebral circulation rarely causes syncope.

Neurogenic (vertebrobasilar insufficiency)

Px: FND

Dx:

U/S

CT/MRA

How well did you know this?
1
Not at all
2
3
4
5
Perfectly
55
Q

Psychiatric disorder (eg, anxiety, depression, conversion disorder)

A

A high incidence (24%–35%) of Psychiatric disorders has been reported in patients with syncope.

Px: In patients faking it, preform the Face-palm maneuver

How well did you know this?
1
Not at all
2
3
4
5
Perfectly
56
Q

Seizure disorder

A

A history of a seizure disorder may be present. Additional findings include cyanosis or absence of pallor during the episode, frothing at the mouth, 👅tongue biting, disorientation, postictal muscle aching and somnolence, age younger than 45 y, and duration of unconsciousness >5 min. Diaphoresis or nausea before the event and postsyncopal orientation argue against seizure.

Evidence of tongue biting is the most reliable finding to differentiate epileptic seizure and syncope. It has low sensitivity (33%) but high specificity (96%) for epileptic seizure (ie, very few false positives), making it highly useful in confirming the diagnosis when present. Lateral tongue biting is likely even more specific for epileptic seizure as frontal tongue biting is occasionally seen with syncope.

  • Post-Ictal state
  • Urinary incontince
  • Residual confusion
57
Q

Subclavian Steal

A

The subclavian steal syndrome results from subclavian or innominate artery stenosis that causes retrograde blood flow in the vertebral artery, diverting flow from the brainstem
and producing hypoperfusion. The degree of subclavian artery stenosis that produces symptoms is variable, but even minor (~40%) stenosis may sometimes do so. A difference between blood pressures measured in the two arms is nearly always found, the average difference being a 45 mm Hg decrease in systolic pressure in the arm supplied by the stenotic vessel. Symptoms of distal limb or vertebrobasilar ischemia may occur. Cerebrovascular risk factors should be modified; stroke is rare. Arteriography and
revascularization procedures may be considered.

58
Q

Infective Endocarditis

A

Hx: Injection drug use, recent procedures associated with risk of transient bacteremia, presence of a prosthetic valve, and certain cardiac abnormalities. Rheumatic fever

Px: New cardiac murmur, new-onset heart failure, focal neurologic signs, splenomegaly, and cutaneous manifestations (eg, petechiae, splinter hemorrhages). The presence of Osler nodes (violaceous, circumscribed, painful nodules found in the pulp of the fingers and toes) or Janeway lesions (painless, erythematous, macular lesions found on the soles and palms.

Mitral valve disease, usually mitral valve prolapse with coexisting mitral 💦 regurgitation, is the most common valvular abnormality detected in patients with infective endocarditis.

About 75% of patients with IE have previously damaged heart valves, with mitral valvular disease being the most common.

Aortic root disease (AR), the characteristic early 💎 diastolic murmur is best heard along the right sternal border; in contrast, when AR is due to valvular disease, the early diastolic murmur is best heard along the left sternal border (3rd and 4th intercostal spaces); cardiac conduction abnormalities are more common with aortic valve than with TV involvement

The aortic valve is the second most common cardiac valve involved in IE, usually in patients with prior history of congenital bicuspid aortic valve with associated aortic stenosis.

Tricuspid valve (TV) is a frequent site of endocarditis among patients with 💉 intravenous drug use. TV endocarditis usually presents with a holosystolic** murmur of tricuspid regurgitation** that becomes accentuated with inspiration

Dx: Leukocytosis, normocytic normochromic anemia, electrocardiographic conduction defects (atrioventricular block from extension of infection into the conduction system), hematuria, and low serum complement levels (eg, glomerulonephritis). CRX suggesting heart failure or septic emboli from right-sided endocarditis (eg, multiple bilateral small nodules on chest radiograph) raise suspicion for infective endocarditis.

Laboratory data include an elevated erythrocyte sedimentation rate (ESR). The ECG shows evidence of first-degree AV block. An antistreptolysin O antibody is necessary to document prior streptococcal infection.

Evidence of recent streptococcal infection plus two major manifestations or one major and two minor manifestations satisfy the Jones criteria for diagnosis of acute rheumatic fever.

Major criteria include ❤ carditis, polyarthritis, s. chorea, erythema marginatum, and subcutaneous nodules.

Minor manifestations include fever, polyarthralgia (Joints) elevated erythrocyte sedimentation rate, and PR prolongation on ECG.

💙 DUKE

Definite endocarditis = 2 MAJOR criteria OR 1 major + 3 minor criteria OR 5 minor criteria

Possible endocarditis = 1 major + 1 minor criterion or 3 minor criteria

DUKE MAJOR Criteria for Dx:

  1. Microbiologic (Bacteremia) - any of the following:

Typical microorganisms (including Staphylococcus aureus) grown from two blood cultures

A microorganism grown from persistently positive blood cultures

Positive serologic test or single positive blood culture for Coxiella burnetii (Q fever) and for Bartonella, Legionella, Brucella, Mycoplasma, and Chlamydophila.

Additional causes of culture-negative endocarditis include a group of gram-negative pathogens constituting the HACEK group:(Haemophilus, Actinobacillus, Cardiobacterium, Eikenella, and Kingella species), nutritionally variant streptococci (eg, Abiotrophiaspecies, Trophermyma whippelii), and fungi (eg, Aspergillus species, Histoplasma capsulatum).

IE due to E corrodens is usually seen in the setting of poor dentition and/or periodontal infection, along with dental procedures that involve manipulation of the gingival or oral mucosa.

  1. Evidence of endocardial involvement (either of the following):

Transthoracic echocardiogram (TTE):oscillating intracardiac mass, abscess, or new partial dehiscence of a prosthetic valve(vegetation). *Transesophageal echocardiography (TEE) is the[initial test of choice]when there is a moderate or high pretest probability of endocarditis (eg, in patients with staphylococcal bacteremia or fungemia, a prosthetic heart valve, or an intracardiac device).TEEis the initial imaging test in some clinical situations, such as detection of left atrial thrombus, evaluation of prostheticmitral valve dysfunction, and evaluation of suspected aortic dissection, as well as in patients with a moderate to high pretest probability of endocarditis.

Physical examination: new valve regurgitation (change in pre-existing murmur is not sufficient). The vegetations of IE usually cause regurgitant murmurs.

DUKE MINOR Criteria:

  1. Predisposing heart condition (RF, IVDA, endo, prosthetics) or 💉injection drug user
  2. Fever: Body temperature >38.0°C (100.4°F)
  3. Vascular phenomena: major arterial emboli, septic pulmonary infarcts, mycotic aneurysm, intracranial hemorrhage, conjunctival hemorrhage, or Janeway lesions
  4. Immunologic phenomena (Rheumatologic): glomerulonephritis, Osler nodes, Roth spots, positive rheumatoid factor
  5. Microbiologic: serologic evidence of infection or positive blood cultures not meeting the major criteria (a single blood culture for coagulase-negative staphylococci is not sufficient)

Staphylococcus aureus

  • Prosthetic valves
  • Intravascular catheters
  • Implanted devices (eg, pacemaker/defibrillator)
  • Intravenous drug users

🌴Viridans group streptococci ( Streptococcus sanguinis 💦 belongs to the viridans group of streptococci, which also includes S mitis, S oralis, S mutans, S sobrinus, and the S milleri group)

  • Gingival manipulation
  • Respiratory tract incision or biopsy

Staphylococcus epidermidis

  • Prosthetic valves
  • Intravascular catheters
  • Implanted devices

Enterococci**

  • Nosocomial urinary tract infections

Streptococcus gallolyticus (S bovis) or Clostridium septicum

  • Colon carcinoma
  • Inflammatory bowel disease

Fungi (eg, Candida species)

  • Immunocompromised host
  • Intravascular catheters
  • Prolonged antibiotic therapy

💊 Rx: Consider antibiotic prophylaxis only in patients with: High-risk cardiac conditions include the presence of a prosthetic cardiac valve, unrepaired cyanotic congenital heart disease, congenital heart disease repair with prosthetic material or device within the last 6 months, presence of palliative shunts and conduits, cardiac valvulopathy in cardiac transplant recipients, and a history of infective endocarditis. High-risk surgical procedures include dental procedures involving manipulation of gingival tissue or the periapical region of teeth or perforation of the oral mucosa as well as respiratory tract procedures that involve perforation of the respiratory mucosa (tonsillectomy, adenoidectomy).

Amoxicillin and cephalosporins, such as cephalexin, are frequently prescribed. Clindamycin, azithromycin, or clarithromycin are appropriate alternatives in patients with penicillin allergy.

Tx: Empiric therapy for early prosthetic valve infective endocarditis includes vancomycin, gentamicin, and rifampin for multidrug-resistant bacteria, particularly coagulase-negative staphylococci.

The Infectious Diseases Society of America Clinical has practice guidelines that recommend intravenous vancomycin (or daptomycin) for the treatment of methicillin-resistant Staphylococcus aureus (MRSA) infective endocarditis. The recommended duration of treatment for S. aureus-associated native valve infective endocarditis is 6 weeks.

Cx: ❗

Septic pulmonary emboli occur in up to 75% of patients with tricuspid endocarditis, most commonly due to Staphylococcus aureus in IVDU with IE. Imaging may show pulmonary septic emboli as pulmonary infiltrates, abscesses, infarction, pulmonary gangrene, or cavities.

Symptoms suggesting septic emboli in patients with tricuspid valve endocarditis include shortness of breath, chest pain, and cough. Blindness, focal weakness, localized back or flank pain, hematuria, and gangrenous skin lesions may be embolic manifestations of left-sided infective endocarditis.

Splenic abscess is typically due to infective endocarditis (or seeding from another site of infection). Likely mechanisms include hematogenous seeding or septic emboli to the spleen. Common causative organisms are Staphylococcus, Streptococcus, and Salmonella. Antibiotics alone for treating splenic abscess have a high mortality (up to 50% in some studies). As a result, splenectomy is recommended for all patients. Percutaneous drainage may be an option in poor surgical candidates.\

Perivalvular abscess: Conduction abnormalities on ECG should raise suspicion extending into adjacent cardiac conduction pathways; the conduction block can potentially lead to syncope.

59
Q

Amyloidosis

A

Hx: Neuropathy; dyspnea, right-sided heart failure, restrictive ventricular filling,

Dx: Marked proteinuria, and hepatomegaly disproportionate to other signs of right-sided heart failure.

Characteristic echocardiographic features of amyloidosis include increased ventricular wall thickness, thickened atrioventricular valves, a thickened atrial septum, and pericardial effusion.

The combination of low voltage on electrocardiogram and thick ventricular walls on echocardiogram suggests amyloidosis (or another infiltrative process).

60
Q

Sarcoidosis

A

Bilateral hilar lymphadenopathy with or without pulmonary reticular opacities and skin, joint, or eye lesions cardiac involvement is suggested by the presence of arrhythmias, conduction blocks, or heart failure.

61
Q

Ddx: Restrictive Cardiomyopathy

A

Amyloidosis

Sarcoidosis

62
Q

Peripheral Vacular Disease (PAD)

A

Tx:

Exercise therapy for a minimum of 12 weeks, with 30-45 minutes of exercise at least 3 times a week, is recommended for all patients with claudication. The goal is to reproduce the claudication symptoms during each session. Supervised exercise programs have been shown in multiple clinical trials to reduce symptoms and improve the maximum walking distance in these patients.

Cilostazol should be considered in patients who have persistent symptoms despite antiplatelet therapy and adequate supervised exercise programs.

Cx: Multiple studies assessing the natural history of patients with PAD have found intermittent claudication as a strong predictor of the future risk of 💗 cardiovascular morbidity and mortality. Patients with PAD and intermittent claudication have an estimated 20% 5-year risk of nonfatal myocardial infarction and stroke and a 15%-30% 5-year risk of death due to cardiovascular causes. The risk rises exponentially with progression of PAD, with an estimated 25% 1-year risk of cardiovascular mortality in patients with critical limb ischemia. The presence of PAD is a coronary artery disease risk equivalent, and these patients should be managed with aggressive risk factor modification for prevention of cardiovascular morbidity and mortality. Tx: Counseling for smoking cessation, lipid-lowering therapy, and evaluation and treatment for hypertension and diabetes mellitus.

63
Q

❗ Acute arterial occlusion (acute limb ischemia)

A

❗ Acute arterial occlusion (acute limb ischemia):pain,pallor,poikilothermia (cool extremity),paresthesia,pulselessness, andparalysis(6 Ps).

Acute arterial occlusion can result from 1 of 3 major causes: EMBOLUS from a cardiac or arterial source, arterial thrombosis, or iatrogenic or direct blunt trauma to the artery.

The sudden development of symptoms in a previously asymptomatic patient is most consistent with an embolic occlusion. The majority of arterial emboli originate from the heart, and lower extremities are affected much more commonly than upper extremities. Potential cardiac sources of emboli include left atrial thrombus due to atrial fibrillation, left ventricular thrombus following an anterior myocardial infarction, infective endocarditis (ie, septic emboli), and thrombus from prosthetic valves.

Dx:

Ankle-brachial index (ratio of systolic pressure in ankle to that in arm) is often used as a screening and/or diagnostic tool in patients with suspected peripheral arterial disease (PAD).

Arterial Doppler study or duplex ultrasonography can identify the presence and location of acute arterial occlusion

Tx: ❗ Patients with suspected acute arterial occlusion leading to an immediately-threatened limb (sensory loss, rest pain, muscle weakness) should be immediately started on anticoagulation while further diagnostic procedures are performed.

Heparin prevents further thrombus propagation and thrombosis in the distal arterial and venous circulation.

Symptoms often give important clues as to the likely site of peripheral arterial disease.

Aortoiliac disease: Hx: Buttock and hip claudication, diminished femoral pulses, and erectile dysfunction: referred to as Leriche syndrome.

Popliteal artery disease: Hx: Pain within the lower calf. Tx: Managed primarily with an exercise program and medical therapy. Patients who do not benefit from such conservative management should be considered for femoral-popliteal bypass.

Superficial femoral artery disease: Hx: Produces effort-related discomfort in the upper calf. Tx: Angioplasty may be appropriate for patients who have symptoms related to the superficial femoral artery and have not benefited from medical therapy or who are extremely limited in activity due to ischemia despite medical therapy.

Common femoral arterial occlusive disease: Hx: Thigh pain with effort. Tx: Because of the location of the common femoral artery with respect to the hip joint, surgical therapy or angioplasty, but not stenting, would be considered as part of therapy.

Cx: Multiple studies assessing the natural history of patients with PAD have found intermittent claudication as a strong predictor of the future risk of 💔cardiovascular morbidity and mortality. Patients with PAD and intermittent claudication have an estimated 20% 5-year risk of nonfatal myocardial infarction and stroke and a 15%-30% 5-year risk of death due to cardiovascular causes. The risk rises exponentially with progression of PAD, with an estimated 25% 1-year risk of cardiovascular mortality in patients with critical limb ischemia. The presence of PAD is a coronary artery disease risk equivalent, and these patients should be managed with aggressive risk factor modification for prevention of cardiovascular morbidity and mortality.

64
Q

Vascular disease (HTN) and vasculitis

A

The clinical presentation depends on the type of blood vessels involved (small, medium, or large).

Dx: Patients with small-vessel disease often have hematuria, proteinuria, and an associated systemic illness. Patients with vasculitis can present with rapidly progressive glomerulonephritis.

Hx: Hypertension is an example of medium-vessel disease and is the second leading cause of CKD in the United States. Hypertensive disease generally is slowly progressive, leading to stage 5 CKD in a minority of patients. Blacks have more aggressive CKD caused by hypertension. Renal artery stenosis is an example of large-vessel disease.

65
Q

Wegener granulomatosis

Microscopic polyangiitis

pauci-immune glomerulonephritis

Goodpasture syndrome.

MPGN

SLE.

A

c-ANCA (cytoplasmic ANCA) is more specific for Wegener granulomatosis. p-ANCA (perinuclear ANCA) is specific for microscopic polyangiitis or pauci-immune glomerulonephritis.

If the history and physical examination suggest an underlying thrombotic or vasculitic process or the urinalysis shows erythrocytes, erythrocyte casts, or proteinuria suggesting glomerular disease, obtain assays for antinuclear antibodies, cytoplasmic and perinuclear antineutrophil cytoplasmic antibodies, anti–glomerular basement membrane antibody, hepatitis B surface antigen, hepatitis C antibodies, complement levels, and anti–double-stranded DNA antibodies.

Goodpasture syndrom: Anti–glomerular basement membrane antibody.

MPGN: C3 level is decreased in 60%–70% of patients with type I or type II MPGN.

hepatitis C: Cryoglobulins especially indicated for AKI.

SLE: Anti–double-stranded DNA antibody

66
Q

Granulomatosis with polyangiitis (GPA) (formerly known as Wegener granulomatosis)

A

A small- and medium-sized vessel vasculitis.

Hx: The upper respiratory tract is most commonly involved, resulting in bloody/purulent nasal discharge, oral or auditory canal ulcers, sinusitis, hearing loss, and saddle-nose deformity. Lower respiratory tract involvement may cause dyspnea, cough, and hemoptysis. Systemic symptoms (eg, fever, arthralgia, weight loss, fatigue) are common.

Dx: Laboratory studies often demonstrate mild leukocytosis and anemia. Renal involvement in the form of glomerulonephritis can be delayed from initial presentation and results in hematuria, non-nephrotic-range proteinuria, and renal insufficiency that can be rapidly progressive.

Serum antineutrophil cytoplasmic antibodies (ANCA) should be checked in all patients with suspected GPA (high sensitivity and specificity). Proteinase 3-ANCA (also known as c-ANCA) is most commonly positive in GPA; myeloperoxidase-ANCA (or p-ANCA) is sometimes positive, although it is more commonly positive in other ANCA-associated vasculitis (eg, microscopic polyangiitis, eosinophilic granulomatosis with polyangiitis [EGPA]). HIV can increase the chance of false-positive ANCA results and should be ruled out. Patients with a clinical presentation and ANCA studies consistent with GPA may be initiated on therapy (high-dose corticosteroids and immunomodulators/cytotoxic agents) prior to confirmatory tissue biopsy.

67
Q

Hereditary hemorrhagic telangiectasia (Osler-Weber-Rendu syndrome)

A

An autosomal dominant disorder characterized by diffuse telangiectasias (eg, ruby-colored papules that blanch with pressure), recurrent epistaxis, and widespread arteriovenous malformations (AVMs). In hereditary hemorrhagic telangiectasia, AVMs tend to occur in the mucous membranes, skin, and gastrointestinal tract, but may also be present in the liver, brain, and lung.

AVMs in the lungs can shunt blood from the right to the left side of the heart, causing chronic hypoxemia, digital clubbing, and a reactive polycythemia. Pulmonary AVMs can also present as massive, sometimes fatal, hemoptysis.

NAIL CLUBBING is associated with the following:

  • Lung diseases: lung cancer, chronic bronchitis (not emphysema), TB, bronchiectasis, hypoxemia due to pulmonary shunts
  • GI diseases: IBD (Crohn disease/ulcerative colitis), cirrhosis
  • Cardiac diseases: infective endocarditis, cardiogenic shunts, hypertrophic pulmonary osteoarthropathy
  • Pregnancy
  • Amyloidosis
68
Q

📈 Reading 📈

A

Rate (300, 150, 100, 75, 60, 50, 40, 30, 20) Fast?

Rhythm (P before each QRS; consistent interval)

Axis (I vs. avF)[If L axis, confirm with II]

R axis - R vent. hypertrophy

L axis - Inferior MI, LAFB

Intervals

P wave 80 ms

P-R (120 - 200 ms or 3-5 little boxes)

QRS < 120ms (3 little boxes)

Segments

QT (50 - 500ms or < .5 R-R

QTc mean <430 women <450 men

Hypertrophy

P wave for atrial abnormalities

R wave for ventricular

Infarction, Ischemia, or

Q waves

Inverted T

ST elevation (transmural)/depression (ischemia)[subendocardial]

ST depression

69
Q

Leads

A

🔽 “Inferior” II, III, aVF 🔼 (Reciprocal = aVL)

The right coronary artery (RCA) supplies blood to the right ventricle, the SA node, the inferior portions of the left ventricle, and usually to the posterior portion of the left ventricle and the AV node. Infarctions involving the SA node may produce sinus dysrhythmias, including bradycardia and sinus arrest.

Cx: The use of verapamil or beta-blockers can further worsen the sinus node dysfunction and result in hypotension or shock.

V1-V2

Septal (R ventricle)

LAD

V3-V4

Left Anterior wall

V5-V6

Left Lateral

LCx

V4Rb

Right ventricle

Right coronary artery

aOften associated with inferior and/or lateral ST-elevation infarctions and tall R wave in V1.

bIndicates a precordial lead placed at the V4 position on the right side of the chest.

Dextrocardia in the adult is most often accompanied by situs inversus or right sided malposition of the abdominal viscera. Isolated dextrocardia without abdominal visceral malposition is uncommon in the adult.

  • 1) inverted P waves in 1 and aVL;
  • 2) negative QRS complex and T wave in 1;
  • 3) progressively decreasing voltage across the precordium.
70
Q

Long QT

A

Hypocalcemia, toxins, hypothermia, and many medications may also lead to QT prolongation.

There are several congenital QT prolongation syndromes, with Romano-Ward syndrome being most common. Romano- Ward is characterized by prolonged QT and congenital deafness, and there may be a family history of sudden cardiac death.

71
Q

AV block

A

First Degree: Prolonged P-R interval (> 0.2 second)[200 ms]

Due to delayed impulse transmission at a number of possible locations from the atria to the ventricles. It can occur in the atria, AV node, bundle of His, or the infra-Hisian conduction system (bundle branches or fascicles).

Tx: Most first-degree AV blocks with normal QRS duration occur due to conduction delay in the AV node and require no further evaluation unless there are significant associated bradycardic symptoms (eg, syncope). Although it is considered a benign finding, the presence of first-degree AV block has been associated with a higher risk of heart failure, atrial fibrillation, and overall mortality.

Second Degree: Dropped QRS; not all P waves transmit to produce ventricular contraction

Mobitz type I (Wenckebach) is an intermittent AV nodal block with progressive prolongation of the PR interval until the dropped beat [benign] nonconducted P wave; It may be seen in normal individuals; other causes include inferior MI and drugs that block the AV node (digoxin, beta-blockers, or calcium-channel blockers).

❗ Mobitz type II is due to a block in the his-purkinje system below the AV node, with a constant PR interval and periodic dropped beats. [Tx: 🔌Pacemaker] intermittent “dropped” QRS complex. The QRS complex can be wider than in Mobitz type I.

Third Degree: Complete heart block: Complete AV dissociation; random P wave and QRS

Tx: Complete heart block in the setting of acute myocardial infarction requires temporary (and often permanent) transvenous pacemaker placement. Atropine may be used as a temporizing measure.

Cx:

Lyme carditis can manifest by acute-onset, high-grade atrioventricular conduction defects that occasionally may be associated with myocarditis. Carditis occurs in 5% to 10% of patients with Lyme disease, usually within a few weeks to months after infection. Atrioventricular block can present in any degree, and progression to complete heart block is often rapid. The prognosis is good, usually with resolution of atrioventricular block within days to weeks. The presence of the characteristic skin rash, erythema migrans, with or without a history of tick bite in an endemic region, has a greater than 80% probability of being caused by Borrelia burgdorferi infection and is sufficient to support a decision to treat Lyme disease empirically without laboratory confirmation of the diagnosis. Most cases of Lyme carditis resolve spontaneously, and neither temporary nor permanent pacemaker therapy is needed.

“AV dissociation” is a term used specifically to refer to the type of mechanism where the intrinsic sinus node and AV junction pacemakers transiently get “out of synch” due to either sinus slowing and/or to acceleration of the AV junctional escape mechanism, and not to actual AV conduction block.

Sinus with AV dissociation

Sinus P waves “marching right through” in and out of the QRS supports the presence of two independent pacemakers.

Intermittent sinus “capture” beats.

72
Q

RBBB

A

Complete Right Bundle Branch Block:

  • Wide QRS complex (> 120 msec)
  • RSR’ pattern in lead V1 with a wide terminal R wave
  • wide negative S wave in V6

RBBB may be seen in normal hearts but is usually associated with an underlying pathology. Conditions such as chronic obstructive pulmonary disease (COPD), pulmonary hypertension, atrial septal defect (ASD), or pulmonic stenosis may, in the long run, result in RBBB because of the hypertrophy that occurs in response to pressure or volume overloads. The conduction systems of aging hearts may also undergo degenerative changes, causing RBBB in the absence of any other pathology. Occlusion of the left anterior descending artery (LAD), causing an acute anterior myocardial infarction, can also cause RBBB because the LAD typically supplies the proximal right bundle. The lesions causing RBBB typically occur in the proximal right bundle, though they can also occur at the level of the moderator band and in the terminal portion of the right bundle.

T wave inversions in leads with tall R¢ waves are typical in RBBB and are called secondary changes.

73
Q

LBBB

A

Associated with absent Q waves in leads I, aVL, and V6; a large, wide, and positive R wave in leads I, aVL, and V6;

  • Wide QRS a (prolongation of the QRS complex to greater than 0.12 seconds)
  • RSR’ in V5 or V6

The presentation of acute coronary syndrome with new left bundle branch block should be considered equivalent to ST-elevation myocardial infarction and is an indication for acute reperfusion therapy.

Bundle branch blocks usually produce “secondary” T wave changes such that the ST-T wave vector points opposite in direction of the major vector of the QRS. These secondary ST-T changes are due to the altered sequence of ventricular activation.

74
Q

ST elevations

A

The differential diagnosis of ST elevations not only includes 1) ischemic heart disease (MI, Prinzmetal angina, ventricular aneurysm) but also 2) pericarditis, 3) left bundle branch block (LBBB) (in V1-V3) and 4) normal (“early repolarization”) variant (J point elevation).

In myocardial infarction the ST elevations tend to be localized (inferior, anterior, posterior, lateral), often, but not always with reciprocal ST depressions.

75
Q

LVH

A

By voltage: R wave 12-13 mm in aVL, plus left atrial abnormality (LAA)?

76
Q

T wave inversions

A

The pattern may occur atypically with 💔myocardial ischemia, especially with underlying LVH.

👦🏽Juvenile T waves: T wave inversions in V1 to V2 are common physiologic variants findings, most prevalent in in young adults and women , and more rarely may extend to V3. This benign T wave finding is present in about 10 percent of young adult American women.

🧠 Neurogenic T waves (“CVA T-wave pattern”) are ❗ “dramatic”T wave repolarization abnormalities seen especially withintracranial bleeds. Much more rarely, the findings are present with thrombotic or embolic type CVAs, or with other central nervous system pathologies (not specific for CVA).

77
Q

💔 ACS: UA, NSTEMI, STEMI (Acute coronary syndrome)

A

“Abnormal cardiac biomarkers in the setting of evidence of acute myocardial ischemia”

ACS

Unstable Angina

Dx: 📈 T-wave inversions

🧪 LACK of elevation of cardiac biomarkers ❌

NSTEMI elevated troponin and LACK of ST segment elevation

Type 1: MI caused by acute atherothrombotic coronary artery disease and usually precipitated by atherosclerotic plaque disruption 💥(rupture or erosion).

Type 2: MI consequent to a mismatch between 🚥 oxygen supply and demand.

Type 3​: V FIB 📈; unexpected death

Type 4a MI requires an elevation of 🧪cardiac troponin (cTn) values greater than five times the 99th percentile URL in patients with normal baseline values.

Type 4b: A subcategory of PCI-related MI is stent/scaffold thrombosis.

Type 5: Coronary artery bypass graft surgery (CABG) MI is defined as an elevation of cTn values >10 times the 99th percentile URL in patients with normal baseline cTn values.

Tx: MONA BASH C (attempt to minimize ischemia by addressing both the supply and demand of oxygen within myocardial cells).

<strong>! M</strong>orphine (should be <strong>avoided</strong>) <strong>CRUSADE INIATIVE</strong> [possibility that it acts by interfering with the antiplatelet effect of the P2Y12 receptor blockers] Masks pain too (😷)

<strong>P</strong>otassium and <strong>M</strong>agnesium — There are no clinical trials documenting the benefits of electrolyte replacement in acute MI. We recommend maintaining the serum potassium concentration above 4.0 meq/L and a serum magnesium concentration above 2.0 meq/L.

<strong>O</strong>2 (only if hypoxic)

Nitrates (🧨Ntitroglycerin) [especially upon discharge 🚗]

Nitrates exert their effect by direct vascular smooth muscle relaxation causing systemic venodilation and an increase in peripheral venous capacitance. Their primary anti-ischemic effect is mediated by systemic vasodilation and decrease in cardiac preloadresulting in a decrease in left ventricular end-diastolic and end-systolic volume. This, in turn, leads to a reduction in left ventricular systolic wall stress - which reflects afterload and is proportional to (pressure * radius / thickness) - and a decrease in myocardial oxygen demand, resulting in relief of anginal symptoms.

❗❗❗Asprin 325mg👄“Chewed” [Other NSAIDS should be discontinued OR per rectum

Beta blocker

🎺 β-Blockers A cardioselective beta-blocker, such as metoprolol, is frequently given in the immediate management of ACS to decrease myocardial oxygen demand, limit infarct size, reduce pain, and decrease the risk of ventricular arrhythmias. Also reduce mortality and should be given to all patients with acute coronary syndrome except those with heart failure, systolic blood pressure of < 90 mm Hg, bradycardia (<50/min), or second-degree atrioventricular block. Complete β-blockade typically results in a resting pulse rate of approximately 55 to 60/min. [Unknown efficacy in NSTEMI] Avoid: Hemodynamic instabillity

<u><strong>A</strong></u>ce (best if <strong>EF < 30</strong>)

❗❗❗Statin (high dose): Atorvastatin, 🅰40 mg to 80 mg/day, or rosuvastatin, 20 mg to 40 mg/day. Cx: Statin associated myositis

Heparin (Antithrombotic therapy) IV or LMWH (enoxaparin)[sub q]

Clopidogel (antiplatelet therapy) loading dose of 300 to 600 mg, then 75 mg po daily. Drug Eluting Stent - 1 yr📅; Bare 📆 Metal - 1 mo; [Thienopyridine derivatives (eg, clopidogrel, ticlopidine, prasugrel)] “Plavix load? CABG VS STENT

Aspirin alternatives, but significantly more expensive. Improve outcomes in patients with recent ACS or stent placement. In patients with stable CAD, thienopyridine derivatives do not improve outcomes.

STEMI

Hx: Frequent but not always exertional chest pain that is often NOT sharp or positional and radiates to both arms. Pain NOT easily reproducible. Likelyhood ratios:

  • Chest pain radiating to the left arm: 2.3
  • Chest pain radiating to the right arm: 2.9
  • Chest pain associated with nausea or vomiting: 1.9
  • Chest pain associated with diaphoresis: 2.0
  • Pleuritic chest pain: 0.2
  • Women and patients with diabetes are more likely to have atypical angina symptoms, such as fatigue, dyspnea, nausea/vomiting.

Px: An S3 is occasionally present.

Dx: 📈 ECG changes or elevated cardiac enzymes in initial workup followed by stress testing or catheterization:

ST-segment elevations in two or more Contiguous leads

≥0.1 mV at J-point for most leads (all ages)

≥0.15 mV for V2-V3 for women

≥0.2 mV for V2-V3 for men ≥ 40 y

≥0.25 mV for V2-V3 for men <40 y

ST-segment depression (indicates true posterior wall MI)

New LBBB

Q waves (infarct): Negative deflection at beginning of QRS. One millimeter (one small box) wide, which is .03 sec. in duration 0.1 mV OR 1/3 the amplitude (or more) of the QRS complex deep. Old infarcts: significant Q waves (like infarct damage) remain for a lifetime. OR QS complexes in I, II, aVL or V4-V6 in 2 contiguois leads OR Q wave in V2-V3 > .02 sec OR QS in V2 and V3 OR R wave > .04 sec in V1-V2 and R/S > 1 with a w wave.

It is important to remember that up to 20% of patients with acute coronary syndrome can have a normal ECG.

🧪 Cardiac troponin T or troponin I peak at 3 to 6 hours and remain elevated for days. Cardiac troponins, and particularly troponin T, may be elevated in patients with kidney failure.

Telemetry monitoring w/ ongoing ECG assesments.

Coronary angiography allows direct evaluation of the coronary anatomy, with possible percutaneous coronary intervention or surgical revascularization if indicated.

Tx: Patients with ischemic chest pain and STEMI benefit from reperfusion therapy:

🔪 Primary angioplasty (primary percutaneous coronary intervention)[PCI] [The goal of therapy in patients STEMI is to perform PCI within:

  • ⌚ 9️⃣0️⃣ minutes of presentation (first medical contact to device time) to a PCI-capable facility OR
  • 120 minutes if the patient requires transfer from a non–PCI-capable hospital (to allow time for transport to a PCI-capable facility).
  • However, patients may benefit from treatment up to 12 hours after the onset of symptoms and possibly even after this time, depending on clinical circumstances].
  • Fibrinolysis is recommended for STEMI patients arriving within 12 hours of symptom onset who cannot undergo PCI (eg, PCI is not available within the recommended time frame). Thrombolytic therapy has NOT shown a clear benefit for patients with STEMI who present more than 12 hours after the onset of symptoms. It remains second-line treatment when PCI is not possible or is contraindicated.

Cx: Stent thrombosis is a potentially fatal complication of coronary artery stenting, and long-term dual antiplatelet therapy with aspirin and platelet P2Y12 receptor blocker (clopidogrel, prasugrel, ticagrelor) is recommended to reduce the risk of stent thrombosis after intracoronary drug-eluting stent placement. Premature discontinuation of antiplatelet therapy is the strongest predictor of stent thrombosis within the first 12 months, and all patients should be screened for, and counseled regarding, medication compliance to reduce the risk of stent thrombosis.

TIMI score:

Cx:

Post-myocardial infarction (MI) complications:

Re-infarction: Hours - 2 days

Ventricular septal rupture💔: Typically occurs 3- 5 days following a large MI involving the left anterior descending artery or right coronary artery. Communication between the left and right ventricle leads to increased pressure and volume load on the right ventricle and results in acute right-sided and left-sided heart failure. Patients develop acute cardiogenic shock (eg, pulmonary edema, hypotension, confusion due to poor organ perfusion), and demonstrate jugular venous distension and hepatomegaly due to liver congestion. In addition, blood flowing from the left to right ventricle creates a harsh holosystolic murmur with palpable thrill 🎉that can be appreciated at the left sternal border. The diagnosis of interventricular septum rupture can be confirmed by pulmonary artery catheterization detecting a left-to-right shunt with a step-up in oxygen saturation from the right atrium to right ventricle or by transthoracic echocardiography with visualization of the septal defect.

Free wall rupture: Hours to 2 weeks: Typically occurs between 5 days and 2 weeks following a large MI. Patients develop pericardial tamponade due to hemopericardium leading to rapid decompensation and death.

A large pericardial effusion should raise suspicion for LV free wall rupture, which usually causes cardiac tamponade (eg, dyspnea, hypotension, pulsus paradoxus, elevated jugular pressure) and can progress rapidly to pulseless electrical activity.

Post-infarction angina: Hours to 1 month

Papillary muscle rupture is a mechanical complication typically occurring 2-7 days post MI, often presents dramatically with acute, severe MR (eg, hypotension, pulmonary edema, cardiogenic shock) and does not cause persistent ST-segment elevation.

Ventricular aneurysm (VA): 5 days to 3 months post MI: The healing process following MI results in replacement of necrosed myocardium with fibrous scar tissue, which in the case of a large MI can result in convexity of a large portion of the left ventricular wall (most commonly in the perfusion field of the left anterior descending artery). The aneurysm is dyskinetic with the remaining healthy ventricular wall, resulting in impaired ejection fraction. ECG typically demonstrates persistent ST-segment elevation and deep Q waves in the leads corresponding to the previous MI, and diagnosis is confirmed by echocardiogram showing a thinned and dyskinetic myocardial wall. Large VAs can lead to progressive left ventricle (LV) enlargement, causing heart failure, refractory angina, ventricular arrhythmias, mural thrombus with systemic arterial embolization, or mitral annular dilation with mitral regurgitation (MR) Diagnosis is confirmed by echocardiography (thinned, dyskinetic LV portion in the area of prior MI).

Acute pericarditis can present in the first several days after MI, can cause diffuse ST-segment elevation.

Postcardiac injury syndrome (Dressler syndrome), an immune-mediated pericarditis, can occur weeks to months post MI.

78
Q

Cardiac Catheterization ❗complications

A

Hematoma (mass, no bruit)

Patients with hematoma present with localized discomfort and/or swelling of the soft tissue. If the arterial puncture site is above the inguinal ligament, the hematoma can extend into the retroperitoneal space, even with minimal visible localized hematoma, and present with sudden hemodynamic instability and ipsilateral flank or back pain. Diagnosis is confirmed with non-contrast CT scan of abdomen and pelvis or abdominal ultrasound. Treatment is usually supportive, with intensive monitoring, bed rest, and intravenous fluids or blood transfusion. Surgical repair of hematomas or retroperitoneal hemorrhage is rarely required. Since local hematoma formation is not uncommon, patients should be advised to avoid strenuous activity or lifting heavy objects for one week post catheterization. Radial artery approach leads to fewer local vascular complications, which has led to an increasing number of cardiac catheterizations being performed via this approach.

Pseudoaneurysm (pulsatile mass, systolic bruit)

Arteriovenous Fistula (no mass, continuous bruit).

The femoral vein can be inadvertently punctured during needle insertion attempts to obtain femoral arterial access. Following the procedure, inadequate hemostasis may allow persistent bleeding from the arterial puncture site to track into the venous puncture site, creating an AVF. Most patients are initially asymptomatic but gradually develop mild swelling and localized tenderness. As arterial pressure exceeds venous pressure throughout the cardiac cycle, a continuous bruit with a palpable thrill is typically present. In addition, distal pulses may be diminished in the affected extremity. An untreated AVF can progressively enlarge and lead to limb edema (due to venous hypertension), limb ischemia (due to redirection of arterial blood flow), and high-output heart failure (due to blood returning to the right atrium without passing through peripheral resistance). The diagnosis is typically confirmed by duplex ultrasound. Management of small AVFs involves observation (sometimes resulting in spontaneous closure) or ultrasound-guided compression. Large AVFs typically require surgical repair.

Cholesterol crystal embolism

Aortic atherosclerotic plaques occur in patients with risk factors for atherosclerosis (eg, hypertension, hypercholesterolemia, diabetes) and can lead to systemic emboli (cerebral, visceral, or lower extremity). Embolism can happen spontaneously, but occurs more commonly during vascular procedures such as peripheral angiography or interventions, guidewire or catheter manipulations during cardiac catheterization, and intraaortic balloon pump insertion. Hx: The diagnosis should be suspected in patients with a constellation of acute/subacute renal failure, typical skin manifestations, and/or gastrointestinal symptoms (eg, nausea, abdominal pain) after a recent vascular procedure. Livedo reticularis is a reticular, erythematous or purple discoloration of the skin that blanches when pressure is applied; in patients with cholesterol emboli, it is typically seen bilaterally in the lower extremities (due to embolization to the vessels). Laboratory studies may show eosinophilia/uria (due to interleukin-5 activation) as well as hypocomplementemia (likely due to neutrophil activation) in cases of recent atheroembolism. Urinalysis is typically benign, with few cells or casts. When the diagnosis remains unclear, skin or renal biopsy may be performed for confirmation.

79
Q

inferior wall STEMI

A

inferior wall STEMI (typically due to occlusion of the right coronary artery) has up to a 50% chance of involving the right ventricle. Clear lungs on auscultation further suggest right ventricular MI (RVMI).

RVMI leads to impaired RV filling and often creates high sensitivity to intravascular volume depletion (a phenomenon not observed in pure left ventricular MI).

❌The administration of nitrates (eg, sublingual nitroglycerin) should be avoided as venous dilation causes an abrupt decrease in RV preload and can lead to profound hypotension. Similarly, diuretics (volume depletion) and opiates (venous dilation) should be avoided or administered with caution.

Cx: In patients with a suspected RVMI, hypotension and low or normal jugular venous pressure (<3 cm H2O above the sternal angle at 30-45 degrees recumbency) strongly suggest cardiogenic shock due to inadequate RV preload. Tx: These patients require bolus with isotonic saline to increase RV preload and improve cardiac output.

Tx: Otherwise, patients with RVMI should undergo standard management of an acute STEMI with dual antiplatelet therapy, statins, anticoagulation, and urgent revascularization (eg, primary percutaneous intervention). Beta blockers (eg, metoprolol) are sometimes appropriate but are contraindicated in patients with bradycardia or cardiogenic shock.

80
Q

Sick sinus syndrome (SSS)

A

Inability of the sinoatrial node to generate an adequate heart rate.

Age-related degeneration of the cardiac conduction system with fibrosis of the sinus node is the most common cause.

SSS comprises a collection of pathologic findings that result in bradycardia.

Fibrosis may also affect the atria, leading to paroxysmal atrial arrhythmias such as atrial fibrillation or bradycardia-tachycardia syndrome (bradycardia alternating with supraventricular tachycardia).

“Junctional escape” = narrow QRS vs “Ventricular escape” = wide QRS

Dx: ECG typically shows sinus bradycardia, sinus pauses (delayed P waves), and sinoatrial nodal exit block (dropped P waves), and exercise testing demonstrates chronotropic incompetence (inadequate heart rate response to exercise).

Tx: Definitive management for SSS requires placement of a pacemaker. Once a pacemaker is placed, rate-control medications (eg, beta blockers) can be administered in patients with persistent paroxysmal tachyarrhythmias.

81
Q

Pericarditis

A

The ST elevations (ventricular current of injury from associated epicarditis) are due to a ventricular current of injury from the pericardial inflammation. The ST elevations in pericarditis are due to a superficial ventricular current injury associated with the pericardial inflammation and the PR segment displacement has been attributed to a superficial epicardial or sub-epicardial atrial current of injury. Sinus tachycardia is a common finding due to pain and, less often, pericardial effusion/tamponade.

Dx: ST-segment elevation (diffuse) ST elevation in all limb leads (except for the maverick, aVR) and in the precordial leads. More specifically (less commonly) PR-segment depression from an atrial current of injury. that eventually evolves to diffuse T wave inversion. The characteristic PR-segment depression (opposite in polarity to the ST segment) due to a concomitant atrial injury current.

Distinction between acute pericarditis and normal variant early repolarization can be difficult. Precordial ST elevations in the latter are usually oriented more toward leads V3-4, while in acute pericarditis (as is case here) ST elevations persist out to V5-V6, more laterally. The presence of PR deviations (up in aVR and down in the infero-lateral leads) with pericarditis is also often helpful.

Tx: 🧯Anti-inflammatory

Cx: 👗Dressler syndrome (post–myocardial infarction pericarditis), which may occur about 1 to 2 weeks post–myocardial infarction. It is thought to be an autoimmune phenomenon.

82
Q

Anterior Fascicular block

A

LAFB

Pure left anterior fascicular block (or hemiblock) causes marked left axis deviation (-45 degrees to -90 degrees), typically with rS complexes in II, III, and aVF, and qR complexes in I and aVL, with a QRS duration of <120 ms.

83
Q

DDx: Supraventricular Arrhythmias

A
84
Q

Sinus Tachycardia

A

Hx: Fever, exercise, anxiety, pain, anemia, thyrotoxicosis, hypoxemia, cocaine use, and alcohol withdrawal are common causes of sinus tachycardia.

Dx: The P waves have normal morphologic features but can become difficult to see with heart rates greater than 140/min because the P waves begin to merge with the preceding T waves.

Slowing the heart rate with carotid sinus massage often shows the hidden P waves and establishes the diagnosis.

85
Q

Premature atrial complexes (PAC’s) or Atrial Premature Beats (APB’s) or Atrial ectopic beat

A

Occur when there is premature activation of the atria originating from a site other than the sinoatrial node. ECG will show an early P wave. PACs by themselves represent a benign arrhythmia that can occur both in healthy individuals and in patients with a variety of cardiovascular and systemic diseases.

They may occur singly or in a pattern of bigeminy. PACs are usually asymptomatic; however, in some patients, they can cause symptoms of “skipped” beats or palpitations. Occasionally they can precede atrial fibrillation. Treatment is required only when symptoms cause distress or when there is supraventricular tachycardia.

Longer PR in the atrial premature complexes?

Frequent atrial ectopy, especially in the post “coronary artery bypass graft (CABG) surgery” setting, may be a forerunner of atrial fibrillation.

Tx: Even in asymptomatic patients, precipitating factors such as tobacco, alcohol, caffeine, and stress should be identified and avoided.

86
Q

Supraventricular Tacycardia’s (SVT):

A

SVT refers to ANY tachycardia originating above the His-bundle and INCLUDES:

  • Sinus tachycardia
  • Multifocal atrial tachycardia (MAT)
  • Atrial flutter
  • Atrial fibrillation
  • Atrioventricular (AV) nodal reentrant tachycardia (AVNRT)
  • AV reentrant tachycardia (AVRT)
  • Junctional tachycardia

Hx: Patients usually present with palpitations, although some may have dizziness, lightheadedness, shortness of breath, diaphoresis, chest pain, presyncope, or syncope.

Paroxysmal supraventricular tachycardias (PSVTs) are SVTs with abrupt onset and offset; they include AVNRT, AVRT, atrial tachycardia, and junctional tachycardia.

Tx: 🎵 Adenosine with its excellent safety profile and extremely short half-life, is the drug of choice for supraventricular tachycardia. The initial dose is 6 mg. A dose of 12 mg can be given a few minutes later if necessary.

🥈🍦Verapamil is the next alternative; if the initial dose of 2.5 to 5 mg does not yield conversion, one or two additional boluses 10 minutes apart can be used.

87
Q

Atrial Tachycardia

A

Commonly occurs in patients with CAD or cor pulmonale.

Px: Heart rate is between 150 and 200

Tx: Atrial tachycardia often terminates without intervention once the underlying cause is treated.

Rx: 🥇First-line drug therapies for stable atrial tachycardia are 🎺β-blockers and 🍦non-dihydropyridine calcium channel blockers (eg, verapamil, diltiazem).

🥈If atrial tachycardia fails to respond to first-line therapy, more advanced antiarrhythmic therapy may be needed with agents such as 👒amiodarone, 🥣flecainide, and sotalol.

Paroxysmal atrial tachycardia (“PAT”) if it occurs paroxysmally

The atrial rate is around 150, and inverted P waves usually follow the QRS complexes because of retrograde atrial conduction from the impulse that starts in the AV node.

88
Q

Multifocal atrial tachycardia (MAT)

A

Hx: Occurs most often in patients with severe COPD or CHF, but it may also occur in patients with pulmonary embolism, congestive heart failure, and hypoxemia.

Dx: Three or more consecutive non-sinus P waves:

Discrete P waves with at least 3️⃣ morphologic patterns with varying P-P, P-R, and R-R intervals and a heart rate between 100/min and 140/min. Best seen in leads II, III, and V1.

Tx: The best treatment is appropriate management of the inciting disturbance; Minimizing or discontinuing agents that may precipitate MAT (such as β-agonist therapy). If MAT persists despite the appropriate treatment of underlying causes, metoprolol or even high-dose magnesium may improve the tachycardia.

89
Q

Atrial flutter

A

The underlying causes of atrial flutter are similar to those for atrial fibrillation and often result from atrial dilation.

Hx: Typical causes include pulmonary embolism, septal defects, mitral or tricuspid valve disease, and chronic left ventricular failure. Atrial flutter may occur in patients without underlying heart disease, however, such as those with thyrotoxicosis or alcoholism.

Dx: Characterized by a saw-tooth pattern on electrocardiogram (lead II) that is most noticeable in the inferior leads.

The atrial rate is between 250 and 300 and is usually associated with a 2:1 or 3:1 AV block, resulting in a “ventricular rate” of approximately 100/min to 150/min.

Grouped beating (“bigeminal”) pattern represents a repetitive variable block mechanism (2:1 and 4:1 conduction).

  • 2:1 Conduction: most common
  • 4:1 every fourth atrial depolarization is conducted through the AV node

With atrial flutter there aren’t P waves per se but “atrial waves”. So if you talk about atrial flutter do not say “the P waves….” since once you say P wave, the informed listener will probably assume it’s not atrial fibrillation or atrial flutter

Tx: Patients with hemodynamic instability due to atrial flutter should be immediately electrically cardioverted. In others, rate control involves the use of a 🍦non-dihydropyridine calcium channel blocker or a 🎺β-blocker. Preventing stroke and cardiac embolization is identical to atrial fibrillation as mentioned previously.

90
Q

Atrial 📈 Fibrillation

A

One of the most common cardiac arrhythmias. The typical electrocardiogram (ECG) presentation of AF includes absent P waves replaced by tiny chaotic fibrillatory waves, irregularly irregular R-R intervals, and narrow QRS complexes.

The pulmonary veins (PVs) are the most frequent location of the ectopic foci that cause AF. Cardiac tissue (myocardial sleeves) extends into the PVs and normally functions like a sphincter to reduce reflux of blood into the PVs during atrial systole. This tissue has different electrical properties than the surrounding atrial myocytes and is prone to ectopic electrical foci and/or aberrant conduction, which can initiate AF. Origination of AF in the PVs is therapeutically useful in patients who cannot achieve rate and/or rhythm control with standard medical therapy. In these patients, the myocardial tissue surrounding the PVs can be disrupted by catheter-based radiofrequency ablation, thereby electrically disconnecting the PVs from the left atrium.

Atrial fibrillation is classified as paroxysmal (lasting <7 days), persistent (lasting >7 days), or long-standing persistent (permanent) (lasting >1 year or associated with failed cardioversion).

Hx: Associated with structural heart disease, such as valvular disease (especially mitral valve disease), dilated cardiomyopathy, hypertension, and coronary artery disease. Heart failure, pulmonary hypertension, and increasing age are also strongly associated. Noncardiac causes include substance abuse (eg, alcohol, caffeine, cocaine, amphetamines), inhaled β-agonists, hypoxemia, COPD, pulmonary embolization, obstructive sleep apnea, and hyperthyroidism. Alcohol excess, sometimes called “holiday heart.” A useful mnemonic for causes of atrial fibrillation is PIRATES: P-pericarditis, pulmonary disease, pulmonary embolism; I-ischemia, infarction, infection, and inflammation; R-rheumatic heart disease; A-atrial septal defect; T-thyrotoxicosis; E-elevated blood pressure, ETOH excess and withdrawal; S-sleep apnea, surgery (cardiothoracic).

Dx: All such patients with new-onset AF should have thyroid-stimulating hormone and free T4 levels measured to screen for occult hyperthyroidism as an underlying cause.

Px: Patients with atrial fibrillation will have a “pulse deficit,” or a higher apical than peripheral pulse rate, because not all ventricular beats are conducted effectively enough to produce a peripheral pulse.

Dx: Disorganized atrial activity at a rate of 350 to 600/min, with no discernible P waves. It is characteristically associated with an irregularly irregular rhythm, and the fibrillatory waves vary in amplitude, morphologic pattern, and interval, creating a rough, irregular baseline between QRS complexes.

The CHADS2 risk score is used to predict the likelihood of stroke in patients with nonvalvular atrial fibrillation.stroke risk factors (ie, Congestive heart failure, Hypertension, A2ge >75 years, Diabetes mellitus, and prior S2troke/TIA/thromboembolism).

0 points: Low stroke risk: aspirin or no treatment is used.

1 Point: Intermediate stroke risk: either warfarin or aspirin is used, based on physician and patient preference.

2 Points or more: high risk: therapeutic anticoagulation is preferred.

CHA2DS2-VASc better recognizes the influence of gender and the presence of established Vascular disease, Age (65-74), Sex)

Tx:

The term 🤠 “lone AF” is occasionally used for patients with paroxysmal, persistent, or permanent AF with no evidence of cardiopulmonary or structural heart disease. Patients with lone AF are generally age <60 and, by definition, have a CHA2DS2-VASc score of 0. The risk of systemic embolization in such patients is extremely low, and anticoagulant therapy is not indicated.

Score of 1: Either anticoagulation or antiplatelet therapy is indicated.

Score of 2 or greater: Oral anticoagulation is recommended for prevention of stroke.

Score of 3 points (hypertension and transient ischemic attack),

Score of 4 points (age range, gender, hypertension, and transient ischemic attack).

The HAS-BLED score can be used to identify patients at high risk of bleeding. Hb decrease by 2; transfusion need. Doesn’t tell us about falls.

Rate control, restoration/maintenance of sinus rhythm, and stroke prevention w/ anticoagulation.

Approximately 50% of episodes of atrial fibrillation convert to a normal rhythm spontaneously.

Rate control approach: patients aged >65 years

🍦Intravenous non-dihydropyridine calcium channel blockers (diltiazem or verapamil)

🎺β-blockers (metoprolol or esmolol).

Warfarin (target international normalized ratio [INR] of 2.0-3.0) reduces the risk of stroke by an average of 64% in patients with nonvalvular atrial fibrillation.

Maintain the INR at 2.0 to 3.0 in patients with nonvalvular atrial fibrillation or at 2.5 to 3.5 in patients with valvular atrial fibrillation.

Rhythm control approach: The antiarrhythmic agents Amiodarone, Flecainide, ibutilide, propafenone, and sotalol are used to maintain patients in sinus rhythm following cardioversion.

If the patient is symptomatic but hemodynamically stable, the initial goal is to reduce the heart rate to 60/min to 110/min with a rate control agent.

❗RVR: Management of atrial fibrillation or atrial flutter with rapid ventricular response (RVR) is determined by the patient’s hemodynamic stability.

❗Rapid (irregular) ventricular response (RVR) AND hemodynamic compromise is treated acutely with electrical synchronized cardioversion (which delivers an electric shock timed to the R wave of QRS complex on the EKG). 120 J

“Cardioversion”: Energy delivery is synchronized to the QRS complex to minimize the likelihood of the shock occurring during repolarization, which can precipitate ventricular fibrillation.

Pulmonary vein catheter radiofrequency ablation or “maze” procedure in those treated unsuccessfully

91
Q

🐺 (AVRT) Atrioventricular Reciprocating Tachycardia [Ventricular preexcitation syndrome]{WPW} w/ tachycardia

A

A bypass tract–mediated reentrant tachycardia in which the anterograde conduction (atrium-to-ventricle) is typically via the atrioventricular node and retrograde conduction is via the bypass tract.

[Associated with an accessory pathway between the atria and ventricle that is not contained within the AV node itself.]

[The aberrant conduction tissue bypasses the normal AV node (hence the PR interval of < 0.20 seconds); it leads the electrical impulse directly to the ventricle (bypassing the His-Purkinje
fibers and widening the QRS complex)]

Dx: Heart rate is typically between 140 and 250

Because bypass tract conduction is typically faster than conduction via the atrioventricular node, atrial activation occurs rapidly after the QRS complex, resulting in a “short R-P” tachycardia, and the P wave is usually located within the ST segment.

“short R-P” tachycardia
A 🎤short PR interval and the presence of a delta wave (a sloping upstroke initiating the QRS complex, which may be wide due to sequential rather than parallel depolarization of the ventricles). This pattern on ECG is referred to as ventricular preexcitation

➕ symptomatic tachycardia (paroxysmal tachyarrhythmias), it is called the 🐺 “Wolff-Parkinson-White syndrome”.

Prolonged QRS duration

Tx: AVNRT may be terminated by maneuvers to increase 💡vagal tone, such as Valsalva or unilateral carotid massage (after careful carotid artery auscultation for bruits).

Rx:

Intravenous 🎵 adenosine, a non-dihydropyridine calcium channel blocker, and β-blockers are often successful in terminating AVRT not responding to vagal maneuvers. Intravenous adenosine has a very rapid onset and is extremely short-acting, with a half-life of 10 seconds, making it an excellent first therapeutic choice. Cx: Adenosine is contraindicated in patients with severe bronchospastic disease.

🎺β-Blockers and 🍦non-dihydropyridine calcium channel blockers can also be used long term to prevent frequent recurrence of AVRT.

In refractory cases, catheter 🔪 radiofrequency ablation is between 95% and 99% successful in preventing AVRT recurrence.

Cx:

Atrial fibrillation (AF) occurs in 10%-30% of individuals with WPW, and is a potentially life-threatening emergency; AF in WPW can bypass the usual rate-limiting function of the AV node, leading to very rapid ventricular response rates. Persistent AF with rapid ventricular response in patients with WPW can ultimately deteriorate into ventricular fibrillation (VF).

Acute treatment of AF in patients with WPW is aimed at prompt control of ventricular response and termination of AF as follows:

Hemodynamically unstable patients require immediate electrical cardioversion

For stable patients, rhythm control with anti-arrhythmic drugs such as intravenous ibutilide or 🤴🏼 procainamide is preferred

92
Q

Atrioventricular nodal reentrant tachycardia (AVNRT)

A

AVNRT is a common type of paroxysmal supraventricular tachycardia (PSVT)

The atria and ventricles are activated simultaneously from a reentrant circuit within the atrioventricular node.

involves a slow pathway and a fast pathway within the atrioventricular node:

The slow pathway conducts slowly but repolarizes quickly.

The fast pathway conducts quickly but repolarizes slowly.

Typical AVNRT (slow-fast) often has an R-P interval so short that the P wave is buried within the QRS complex. The QRS complex is usually narrow as long as conduction below the atrioventricular node is normal.

Accounts for approximately 60% of all SVTs that present as a regular rhythm.

Dx: P wave either is seen just after the QRS complex, which accounts for a short RP interval or is concealed within the QRS complex (no visible P wave)

Tx: Vagal maneuvers (eg, carotid sinus massage, cold-water immersion or diving reflex, Valsalva maneuver, eyeball pressure) increase parasympathetic tone in the heart and result in a temporary slowing of conduction in the AV node and an increase in the AV node refractory period, leading to termination of AVNRT.

Rx: Intravenous adenosine, a non-dihydropyridine calcium channel blocker, and β-blockers are often successful in terminating AVNRT not responding to vagal maneuvers. Intravenous adenosine has a very rapid onset and is extremely short-acting, with a half-life of 10 seconds, making it an excellent first therapeutic choice. Adenosine is contraindicated in patients with severe bronchospastic disease. β-Blockers and non-dihydropyridine calcium channel blockers can also be used long term to prevent frequent recurrence of AVNRT.

In refractory cases, catheter radiofrequency ablation is between 95% and 99% successful in preventing AVNRT recurrence.

93
Q

Ddx: Ventricular Arrhythmias

A
94
Q

Premature ventricular contractions (PVCs) or Ventricular premature beats (VPB’s) or Ventricular premature complexes (VPC’s)

A

PVCs can be identified by the following features:

  • QRS duration >0.12 seconds
  • Bizarre morphology not resembling any conduction abnormality (eg, bundle branch block)
  • T wave in the opposite direction of QRS axis
  • Compensatory pause

“Bigeminy” = PVC to sinus are 1:1

Hx: Rarely have symptoms but may complain of palpitations (symptomatic) or a sensation that the heart has stopped, owing to the post-PVC compensatory pause.

Isolated PVCs should be distinguished from nonsustained ventricular tachycardia (NSVT), characterized by ≥3 consecutive ventricular beats at a rate of >100 beats/min and lasting <30 seconds.

> 10 PVCs per minute in association with decreased LV function have been associated with increased mortality.

Tx: Treatment is based on ameliorating significant symptoms, which may include palpitations, fatigue, and lightheadedness. If the patient’s symptoms are tolerable, no therapy is indicated.

🎺β-blockers (such as metoprolol) 📈 (escalating doses) or calcium channel blockers (CCBs) are first-line therapy. BBs do not directly suppress PVCs; however, they reduce PVC symptoms, possibly by reducing postextrasystolic potentiation (ie, increased systolic contractility following PVCs or other extrasystolic beats).

95
Q

Ventricular tachycardia (VT)

A

Wide-complex tachycardia (QRS duration > 0.120 sec)

Ventricular rate typically ranges from 140/min to 250/min

In a patient with wide-complex tachycardia and a history of coronary artery disease or cardiomyopathy, ventricular tachycardia is the assumed diagnosis.

  • A monophasic wide R wave in V1 (or an Rsr~ or qR-type morphology)
  • Very wide QRS (>140-160 ms,
  • An rS or QS in V6.

Hx: VT often accompanies structural heart disease (prior MI, CAD, cardiomyopathy), most commonly ischemic heart disease, and it is associated with electrolyte disorders (eg, hypokalemia, hypomagnesemia), drug toxicity, valvular heart disease, nonischemic cardiomyopathy, and long QT syndrome.

Dx: Precordial concordance?

Monomorphic VT is usually originates in the vicinity of an underlying ventricular scar.

Nonsustained VT: has ≥ 3 beats but is <30 seconds in duration.

Hx: Usually are asymptomatic but may experience palpitations, dizziness, or syncope.

Sustained VT: Persists >3️⃣0️⃣ seconds or requires termination due to hemodynamic collapse.

Hx: Syncope or near syncope and can also present with sudden cardiac death.

Long QT syndrome is characterized by prolonged ventricular repolarization and a predisposition to the development of polymorphic VT and sudden cardiac death.

Torsades de pointes is a special subset of polymorphic VT where the ventricular rate ranges from 200/min to 300/min. Tx: Magnesium sulfate

Rx:

Sustained:

🍡👒 IV amiodarone has largely replaced other antiarrhythmic agents (sotalol) for hemodynamically stable VT in the acute setting, although lidocaine can be useful in patients with coronary ischemia.

🤴🏽Procainamide and sotalol are also acceptable

🤥Lidocaine can be used as a second-line agent.

Nonsustained: Medical therapy does not improve survival in patients with nonsustained VT, and thus pharmacologic therapy is avoided unless the patient has a history of structural heart disease or long QT syndrome or (rarely) intolerable symptoms.

🎺 β-Blockers are the mainstay of treatment for those with symptomatic nonsustained VT

🍦non-dihydropyridine calcium channel blockers may also be used in patients with structurally normal hearts.

❗Hemodynamically unstable: Epinephrine or vasopressin is recommended for hemodynamic support,

Patients in whom drug therapy for nonsustained VT fails or is not tolerated can be referred for catheter-directed radiofrequency ablation or ICD placement.

Implantable cardioverter-defibrillator (ICD) is an internal defibrillator that senses dangerous cardiac arrhythmias and automatically converts the rhythm to sinus rhythm by either administering a high-energy shock or delivering a short series of paced beats.

An ICD is also indicated in patients with sustained VT in the setting of structural heart disease or when a completely reversible risk factor cannot be identified (such as in patients with hypertrophic cardiomyopathy at increased risk for sudden cardiac death).

ICD placement is also indicated for the primary prevention of sudden cardiac death in patients with (1) NYHA Class II or III heart failure AND an ejection fraction of less than 35% or (2) a prior MI and an ejection fraction of less than 30% (after a waiting period of 40 days).

Catheter-directed radiofrequency ablation of VT is useful in patients with idiopathic VT (those without structural heart disease or another clear etiology) as well as those with frequent recurrences of VT.

Cx: VT is a potentially life-threatening arrhythmia due to rapid, depolarizing impulses originating from the His-Purkinje system, the ventricular myocardium, or both.

96
Q

❗ Ventricular fibrillation (VF)

A

“Ventricular fibrillation (VF) cardiac arrest.” …Fentricular flutter?

Dx: Rate is typically >300/min

Hx: Ventricular arrhythmias (eg, ventricular tachycardia [VT], VF) are common in patients with myocardial ischemia, and VF is the most common cause of sudden cardiac arrest during acute myocardial infarction. More than 50% occur within the first hour of symptom onset. Reentry is the predominant mechanism responsible for ventricular arrhythmias in the immediate post-infarction period.

The underlying mechanism responsible for periinfarction ventricular arrhythmias varies according to the time elapsed since the onset of MI. Arrhythmias occurring within 10 minutes of coronary occlusion are known as “immediate” or phase 1a ventricular arrhythmias. Acute ischemia causes heterogeneity of conduction with areas of marked conduction slowing and delayed activation, which in turn predisposes to reentrant arrhythmias. On the contrary, “delayed” or phase 1b arrhythmias occur about 10-60 minutes after acute infarction and are thought to result from abnormal automaticity.

Tx: In patients with sudden cardiac arrest due to VF, survival depends on early identification, effective cardiopulmonary resuscitation (CPR), and successful defibrillation with return of spontaneous circulation. According to the advanced cardiac life support (ACLS) protocol, all patients with VF or pulseless VT detected on ECG should be managed with immediate defibrillation in an effort to restore a perfusing heart rhythm. ⚡(200 J, then 300, then 360 if using a monophasic defibrillator; 200 J maximum if using a biphasic defibrillator), followed by epinephrine if needed.

“Defibrillation”: (as opposed to cardioversion) provides a high-energy shock at a random point in the cardiac cycle (unsynchronized shock) and is indicated in patients with ventricular fibrillation or pulseless ventricular tachycardia.

Pulseless electrical activity is defined as the presence of electrocardiographic rhythm in the absence of adequate cardiac mechanical contraction to generate a palpable pulse. Asystole is the complete absence of electrical and mechanical cardiac activity.

Reversible causes of asystole/pulseless electrical activity

  • 5 Hs
  • 5 Ts
  • Hypovolemia
  • Tension pneumothorax
  • Hypoxia
  • Tamponade, cardiac
  • Hydrogen ions (acidosis)
  • Toxins (narcotics, benzodiazepines)
  • Hypokalemia or hyperkalemia
  • Thrombosis (pulmonary or coronary)
  • Hypothermia
  • Trauma
97
Q

Accelerated idioventricular rhythm (“slow V tach”)

A

Rate 60 to 100 develops in 25% of post-MI patients. This is a benign rhythm and rarely degenerates into ventricular tachycardia or other serious arrhythmia, so observation is the appropriate choice.

AIVR is classically seen in the context of reperfusion with acute ischemia, but may occur in other settings.

98
Q

Sinus Bradycardia

A

A common rhythm disturbance in acute inferior MI. It is usually caused by increased vagal tone and not by destruction of the SA node. When associated with hypotension, atropine should be given.

99
Q

Dextrocardia

A

Dextrocardia in the adult is most often accompanied by situs inversus or right sided malposition of the abdominal viscera. Isolated dextrocardia without abdominal visceral malposition is uncommon in the adult.

1) inverted P waves in 1 and aVL;
2) negative QRS complex and T wave in 1;
3) progressively decreasing voltage across the precordium.

100
Q

Ddx: Tachycardia

A

Sepsis

Septic Shock

Adrenal Neoplasms:

Pheochromocytoma

Cardiomyopathy:

Dilated Cardiomyopathy

Hypertrophic Cardiomyopathy

Wolff-Parkinson-White Syndrome

Pulmonary:

Thromboembolism

Embolism and Infarction

Gastro:

Esophageal Perforation

Pancreatitis

Acute Pancreatitis

Endocrine:

Hyperthyroidism

Thyroid Storm

Graves Disease

Ob:

Uterine Rupture

Chorioamnionitis

Tachycardia Bradycardia syndrome

Sinus node automaticity is suppressed by the tachyarrhythmia and results in a prolonged sinus pause following termination of the tachycardia.

101
Q

♨♨♨ Hypertension (primary)

A

Dx: Obtain the following studies in all patients: hematocrit, glucose, creatinine, electrolytes, urinalysis, fasting lipid profile, and electrocardiography. Blood or protein in the urine may indicate kidney damage or suggest secondary hypertension.

An electrocardiogram showing left ventricular hypertrophy and/or signs of previous infarction (Q waves) is evidence of cardiovascular damage. Although echocardiography is more sensitive in diagnosing ventricular hypertrophy, it is not routinely recommended in all patients with a new diagnosis of hypertension.

Tx: Weight reduction is most beneficial, and systolic blood pressure can fall from up to 20 mm Hg for each 10 lb of weight lost.

Dietary recommendations to lower BP consist of consuming a diet that emphasizes the intake of fruits, vegetables, and whole grains; includes poultry, fish, legumes, nuts, nontropical vegetable oils, and low-fat dairy products; and restricts the consumption of red meat, sweets, and sugar-sweetened beverages.

DASH (Dietary Approaches to Stop Hypertension) diet can help to achieve these goals.

Dietary sodium reduction: Sodium should be restricted to <2400 mg/day (<1500 mg/day results in greater reductions) or by at least 1000 mg/day. [can lower blood pressure between 8 and 14 mm Hg]

Three to four sessions averaging 40 minutes in duration per week of moderate to vigorous aerobic physical activity may also lower BP. [Less than 10 mmHg]

Rx: In the general adult population <60 years of age, pharmacologic treatment is recommended when the systolic BP is ≥140 mm Hg or the diastolic BP is ≥90 mm Hg. In patients ≥60 years of age, therapy is recommended if the systolic BP is ≥150 mm Hg or the diastolic BP is ≥90 mm Hg. Typically, a single agent decreases systolic BP by 12 to 15 mm Hg and diastolic BP by 8 to 10 mm Hg.

In the general non-African American population, thiazide diuretics, angiotensin-converting enzyme inhibitors (ACEIs), angiotensin receptor blockers (ARBs), and calcium channel blockers (CCBs) may all be considered for initial treatment of hypertension, and all reduce the complications of hypertension.

For African Americans, initial therapy should be a thiazide diuretic or CCB, including those with diabetes.

For all patients (regardless of race or the presence or absence of diabetes) >18 years of age with chronic kidney disease (including those with and those without proteinuria), initial therapy should be an ACEI or ARB because these agents are renoprotective and improve renal outcomes.

In blacks with chronic kidney disease but without proteinuria, the initial agent can be a CCB, thiazide diuretic, ACEI, or ARB. Diuretics may equalize the response of black patients to ACEIs and ARBs. Loop diuretics are preferred for patients with chronic kidney disease and a serum creatinine level >1.5 mg/dL or a glomerular filtration rate <30-50 mL/min/1.73 m2. Thiazide diuretics are much more likely than loop diuretics to cause significant hyponatremia, particularly in elderly women.

In patients with hypertension and ❤ coronary artery disease, 🎺β-blockers are the drugs of choice because they decrease cardiovascular mortality.

The presence of asthma or chronic bronchitis may limit the use of β-blockers.

#1 ACEIs are preferred for use in patients with asymptomatic ventricular dysfunction and symptomatic heart failure because they decrease cardiovascular mortality.

#2 Combination ACEIs and thiazide diuretic therapy reduces recurrent stroke rates.

#3 Compared with β-blockers, ARBs may be specifically beneficial in patients with left ventricular hypertrophy.

🃏 ACEIs and ARBs reduce albuminuria and the progression of chronic kidney disease, including 🍭diabetic nephropathy.

ACEIs produce greater reductions in cardiac morbidity and mortality compared with calcium channel blockers. An increase of up to 33% in serum creatinine is acceptable and not a reason to discontinue ACEI therapy; however, hyperkalemia may limit the use of ACEIs.

Parenteral drugs are usually not necessary in hypertensive urgency, unless symptoms or progressive end-organ damage is present. Initial treatment is with one or more rapid-onset oral anti-hypertensive drugs (such as clonidine or a short-acting ACEI such as captopril) followed by a longer-acting formulation once BP is <180/110 mm Hg. Clonidine may be given hourly until the desired BP is achieved, although care must be taken to not excessively lower the BP. BP should be rechecked within 48 hours.

❗ Avoid short-acting calcium channel blockers in patients with ischemic heart disease (such as nifedipine), because reflex adrenergic stimulation and tachycardia may lead to myocardial ischemia.

Isolated ambulatory hypertension (masked hypertension) 🎭 These patients typically have normal blood pressure readings during clinic visits but their average blood pressure throughout the day and nighttime is elevated. Because detection is difficult, patients often initially present with evidence of hypertensive end-organ damage (eg, retinal arteriovenous nicking consistent with hypertensive retinopathy, increased QRS-complex voltage consistent with left ventricular hypertrophy).

Patients with clinical signs of hypertension but normal blood pressure readings should be tested for masked hypertension with ambulatory blood pressure monitoring (ABPM). A blood pressure cuff is worn throughout a 24-hour period, and blood pressure is measured and recorded at routine intervals (eg, every 20 minutes while awake and every 60 minutes while sleeping). An average 24-hour blood pressure >135/85 mm Hg is diagnostic of hypertension

Resistant hypertension is BP that is not at target level despite maximal doses of 3️⃣ antihypertensive agents, one being a diuretic. Important causes include medication nonadherence, inadequate therapy, excessive alcohol consumption, and other drugs (eg, NSAIDs, sympathomimetic agents).

A patient with resistant hypertension and systolic heart failure should begin taking a newer-generation dihydropyridine calcium channel blocker, such as amlodipine, to improve control of blood pressure (amlodipine and felodipine to have little or no negative inotropic effect (nodal) and a neutral effect on morbidity and mortality rates). However, because they do not decrease morbidity or mortality rates, calcium channel blockers are not first-line treatment for systolic heart failure. Use of calcium channel blockers in systolic heart failure is generally reserved for treatment of conditions such as hypertension or angina that are not optimally managed with maximal doses of evidence-based medications such as ACE inhibitors and β-blockers. Many calcium channel blockers are relatively contraindicated in patients with systolic heart failure because of an associated increased risk of exacerbation of heart failure. Older-generation calcium channel blockers, such as diltiazem, nifedipine, and verapamil, may precipitate exacerbation of heart failure because of their negative inotropic effects.

Cx:

Hypertensive urgency is a severe elevation in BP without acute end-organ damage.

Malignant hypertension is usually seen in patients with long-standing and uncontrolled hypertension. It is associated with retinal hemorrhages, exudates, and/or papilledema.

❗Hypertensive emergency is defined as markedly elevated BP (≥180/120 mm Hg) combined with symptoms or signs of end-organ damage, such as encephalopathy, papilledema, retinal hemorrhages or exudates, stroke, myocardial ischemia or infarction, aortic dissection, pulmonary edema, or acute kidney injury.

Hypertensive encephalopathy is associated with cerebral edema due to breakthrough vasodilation from failure of autoregulation. Patients usually develop insidious onset of headache, nausea, and vomiting followed by non-localizing neurologic symptoms (eg, restlessness, confusion, seizures/coma if untreated). Patients also can develop subarachnoid or intracerebral hemorrhage.

Tx:

🎺Intravenous labetalol is often used in hypertensive urgencies but, as a nonselective beta-blocker, is relatively Cx: asthma

🧨Nitroprusside in the ICU setting is indicated. Cx: Renal insufficiency would be a contraindication.

Intravenous Nitroglycerin, fenoldopam, or enalapril.

102
Q

Pre HTN

A

Although this is not considered a mild form of hypertension, it is a category used to define persons considered at increased risk for the development of true hypertension. Increasing age and family history are also associated with an increased risk of eventually developing hypertension requiring treatment. The patient should be advised to make appropriate lifestyle changes if indicated, such as weight loss (if overweight), increase his exercise level, and control his dietary intake of cholesterol and saturated fats. His blood pressure should then be rechecked in 1 year.

103
Q

SECONDARY HYPERTENSION

Dx: Patients with clinical signs of hypertension but normal blood pressure readings should be tested for masked hypertension with ambulatory blood pressure monitoring (ABPM). A blood pressure cuff is worn throughout a 24-hour period, and blood pressure is measured and recorded at routine intervals (eg, every 20 minutes while awake and every 60 minutes while sleeping). An average 24-hour blood pressure >135/85 mm Hg is diagnostic of hypertension.

A

Dx: Patients initially diagnosed with hypertension should have a detailed history and physical examination. In addition, the following basic testing should be performed:

  • Urinalysis for occult hematuria and urine protein/creatinine ratio
  • Chemistry panel
  • Lipid profile (risk stratification for coronary artery disease)
  • Baseline electrocardiogram (to evaluate for coronary artery disease or left ventricular hypertrophy).

Renal parenchymal disease

  • Elevated serum creatinine
  • Abnormal urinalysis (proteinuria, red blood cell casts)
  • Late manifestations of kidney failure include elevated BUN, creatinine, potassium, and phosphate levels; low calcium level; and anemia. Most patients present at an earlier stage, with minimal signs and symptoms.

Renovascular disease

  • Severe hypertension (>180 mm Hg systolic &/or 120 mm Hg diastolic) after age 55
  • Possible recurrent flash pulmonary edema or resistant heart failure
  • Unexplained rise in serum creatinine
  • Elevation in serum creatinine >30% from baseline after starting angiotensin-converting enzyme inhibitor or angiotensin-receptor blocker
  • Severe hypertension in patients with diffuse atherosclerosis
  • Onset of severe hypertension after age 55
  • Hypertension in a patient with asymmetric kidney size or a small atrophic unilateral kidney
  • Presence of abdominal bruit

Primary aldosteronism

  • Easily provoked hypokalemia
  • Slight hypernatremia
  • Hypertension with adrenal incidentaloma

Pheochromocytoma

  • Paroxysmal elevated blood pressure with tachycardia
  • Pounding headaches, palpitations, diaphoresis
  • Hypertension with an adrenal incidentaloma

Cushing syndrome

  • Central obesity, facial plethora
  • Proximal muscle weakness, abdominal striae
  • Ecchymosis, amenorrhea/erectile dysfunction
  • Hypertension with adrenal incidentaloma

Hypothyroidism

  • Fatigue, dry skin, cold intolerance
  • Constipation, weight gain, bradycardia

Primary hyperparathyroidism

  • Hypercalcemia (polyuria, polydipsia)
  • Kidney stones
  • Neuropsychiatric presentations (confusion, depression, psychosis)

Coarctation of the aorta

  • Differential hypertension with brachial-femoral pulse delay

White Coat

  • Ambulatory blood pressure monitoring is indicated for patients with suspected white coat hypertension, to monitor patients with difficult-to-control blood pressure or those with significant symptoms such as hypotension on therapy, or if autonomic dysfunction is suspected.

Drug-induced

  • Possible causes include NSAIDs, amphetamines, cocaine, sympathomimetic agents (eg, decongestants, dietary supplements), oral contraceptives, antidepressants, bromocriptine, erythropoietin, and glucocorticoids.
104
Q

Renovascular disease (atherosclerotic and fibromuscular [FMD])[Renal artery stenosis]

A

HTN-related symptoms

  • Resistant HTN (uncontrolled despite 3-drug regimen)
  • Malignant HTN (with end-organ damage)
  • Onset of severe HTN (>180/120 mm Hg) after age 55
  • Severe HTN with diffuse atherosclerosis
  • Recurrent flash pulmonary edema with severe HTN

Supportive evidence

  • Physical examination
    • Asymmetric renal size (>1.5 cm)
    • Abdominal bruit
  • Laboratory results
  • Unexplained rise in serum creatinine (>30%) after starting ACE inhibitors or ARBs
    • Imaging results
  • Unexplained atrophic kidney

Renovascular hypertension is the most common correctable cause of secondary hypertension. Renal artery stenosis (RAS) is present in 1% of patients with mild hypertension and in 25%-35% of patients with peripheral arterial disease.

FMD most commonly affects women age 15-50. It is a noninflammatory and nonatherosclerotic condition caused by abnormal cell development in the arterial wall that can lead to vessel stenosis, aneurysm, or dissection. FMD can involve any artery but most commonly involves the renal, carotid, and vertebral arteries.

Px: A systolic-diastolic abdominal bruit can be heard in ~40% of patients; this finding has very high specificity for RAS. Dx: The diagnosis is confirmed by noninvasive assessment with renal duplex Doppler ultrasonography, CT angiography, or magnetic resonance angiography.

  • Possible recurrent flash pulmonary edema or resistant hreat failure

Patients with renal artery involvement usually develop hypertension. Involvement of cerebrovascular arteries (eg, carotid, vertebral) can lead to nonspecific symptoms (eg, headache, pulsatile tinnitus, dizziness) or symptoms of brain ischemia (eg, transient ischemic attack, stroke, amaurosis fugax). FMD can also involve the iliac, subclavian, and visceral arteries.

Renal ischemia (caused by limited blood supply to the poststenotic kidney) induces the activation of the renin-angiotensin-aldosterone system, resulting in sodium and water retention, generalized vasoconstriction, and hypertension. Laboratory abnormalities reflect secondary hyperaldosteronism (ie, hypokalemia, elevated serum bicarbonate). Unlike unilateral RAS, in which the unaffected kidney can typically compensate and the creatinine remains relatively normal, bilateral RAS can result in chronic kidney disease (CKD). Urinalysis is typically bland.

The diagnosis is confirmed with renal imaging (eg, renal ultrasound with Doppler, CT/MR angiography).

Dx: Diagnosis is usually confirmed with noninvasive imaging (computed tomography (digital subtraction) angiography of the abdomen or duplex ultrasound). Patients with inconclusive noninvasive tests require catheter-based digital subtraction arteriography for diagnosis. FMD decreases perfusion to the kidneys, which increases both renin and aldosterone levels (secondary hyperaldosteronism). The aldosterone concentration to renin activity ratio is ~10 (<20).

Digital subtraction angiography (gold standard)[“string of beads”]

ACE inhibitor renal scan

Renal magnetic resonance angiography

Tx:

Management includes blood pressure control, aggressive treatment of atherosclerotic disease (eg, statins, smoking cessation), and ACE inhibitors (with close monitoring of renal function).

🔪Renal revascularization is indicated for most patients with fibromuscular dysplasia, but benefit is less clear with atherosclerotic stenosis.

Percutaneous transluminal kidney angioplasty is indicated for patients with renovascular hypertension secondary to fibromuscular dysplasia, a nonatherosclerotic, noninflammatory renovascular disease.

105
Q

Primary hyperaldosteronism

A

Hx: Difficult-to-control or worsening hypertension despite multiple antihypertensive agents; hypertension at a young age. Characterized by muscle cramping, nocturia, thirst, hypokalemia, and 🧂hypernatremia.

Dx: Spontaneous 🍌hypokalemia, severe hypokalemia after institution of low-dose diuretic therapy. Elevated aldosterone and suppressed renin level, a midmorning ambulatory plasma aldosterone concentration (PAC)/PRA ratio >20 to 30. A ratio greater than 30:1 in conjunction with a
plasma aldosterone concentration of greater than 555 pmol/L (20 ng/dL) has a sensitivity of 90% and a specificity of 91%.

Asses for serum aldosterone suppression after dietary or intravenous sodium loading.

Adrenal CT scan: Bilateral adrenal hyperplasia (also termed idiopathic primary hyperaldosteronism) is the most common cause of primary hyperaldosteronism, followed by unilateral aldosteronoma (Conn syndrome), and more rarely, unilateral hyperplasia or adrenal carcinoma.

Tx: Adrenalectomy is the treatment of choice for aldosteronoma.

Rx: Spironolactone is the treatment of choice for idiopathic hyperaldosteronism, bilateral adrenal hyperplasia, and nonsurgical candidates with aldosteronoma

106
Q

Pheochromocytoma

A

Pheochromocytomas are paraganglioma tumors that arise in the chromaffin cells of the adrenal medulla that accounts for a small number of cases of secondary hypertension (0.1%-0.6%). Ten percent of pheochromocytomas are extra-adrenal, 10% are malignant, 10% recur, and 10% are asymptomatic. The tumors can produce, store, and secrete catecholamines (norepinephrine, epinephrine, and/or dopamine); most produce norepinephrine.

Hx: Hyperadrenergic cyclic spells of hypertension, diaphoresis, palpitations, or headache, familial predisposing syndrome (neurofibromatosis type 1, MEN2, succinate dehydrogenase subunit B mutation), previous vasopressor response to anesthesia or angiography, incidentally discovered adrenal mass, hypertension at a young age (<20 years), drug-resistant hypertension. Other symptoms include anxiety, tremor, and pallor. Chronic complications of excess catecholamine release include cardiac arrhythmias, both dilated and hypertrophic cardiomyopathy, and accelerated atherosclerosis related to hypertension.

Characterized by sweating, heart racing, pounding headache, pallor, tachycardia.

Hypertension is present in more than 90% of patients with pheochromocytoma: more than 50% of these patients have sustained elevations in blood pressure; 30% to 40% have episodic elevations; and up to 10% have no hypertension. Lability of blood pressure is due to episodic catecholamine release, volume depletion, and adrenergic receptor desensitization caused by chronic stimulation.

Dx: Measurement of 24-hour urine catecholamine and metanephrine excretion is used to screen when pretest probability is low. Urinary VMA, Plasma free metanephrine levels are more sensitive and best used when pretest probability is high (if positive, the diagnosis should be confirmed with 24-hour urine studies). Testing should be avoided in the setting of acute illness, as catecholamine production is increased. Patients under marked psychological stress, or with anxiety or panic disorder, may have increased catecholamine production, but measured levels should be less than 2 to 4 times the upper limit of normal.

Non-contrast Adrenal CT or MRI for tumor localization. If no adrenal abnormality is seen, CT scans of the chest, abdomen, and pelvis should be obtained to look for paragangliomas along the sympathetic chain.

Tx: Surgical resection; full α-adrenergic blockade (Phenoxybenzamine, terazosin, prazosin, or doxazosin) prior to surgery to avoid a hypertensive emergency during the procedure.

107
Q

Cushing syndrome

A

Hx: Weight gain, history of recurrent or chronic infections (especially candidal), worsening diabetic control, change in menses, or fractures; feminization, virilization, an abdominal mass (adrenal tumor), or visual field losses (pituitary tumor).

Px: Abnormal fat distribution (not just the presence of adipose tissue), particularly in the supraclavicular and temporal areas, proximal muscle weakness, or wide (>1 cm) purple striae.

Dx: 24-hour urine free cortisol measurement (>3 times normal), overnight 1-mg dexamethasone suppression test (failure to suppress cortisol), or nighttime salivary cortisol level. Typically, 2 to 3 of these tests are performed to confirm the diagnosis; metabollic alkalosis.

Basal ACTH levels <6 pg/mL are found in adrenal forms of Cushing syndrome, while levels >6 pg/mL occur in ACTH-dependent disease. An adrenal computed tomography (CT) or MRI scan should be obtained to localize lesions in patients with suppressed ACTH values. In patients with nonsuppressed ACTH levels, a pituitary MRI scan should be obtained.

Tx: Surgical resection of an identified tumor (adrenal, pituitary, or ectopic) is the optimal therapy for Cushing syndrome. Pituitary radiation (“gamma knife”) therapy can be used for patients with persistent or recurrent Cushing disease after transsphenoidal surgery or for those in whom pituitary surgery is contraindicated.

Rx: Drugs are used adjuvantly in patients undergoing surgery and as sole therapy for those with occult ectopic ACTH secretion or metastatic adrenal cancer to reduce cortisol production. Ketoconazole, mitotane, metyrapone, and aminoglutethimide reduce endogenous cortisol production and reverse most signs and symptoms of Cushing syndrome.

108
Q

Hypothyroidism

A

Hypothyroidism can cause hypertension and typically also presents with other symptoms, including weight gain, fatigue, bradycardia, and dry skin.

109
Q

Primary hyperparathyroidism (PHPT)

A

Excess parathyroid hormone causes hypercalcemia due to increased renal calcium reabsorption, gastrointestinal calcium absorption, and bone resorption. Patients can present with bone pain, kidney stones, gastrointestinal symptoms, and neuropsychiatric symptoms (“bones, stones, abdominal moans, and psychic groans”). The majority (80%) of PHPT cases are due to parathyroid adenoma.

PHPT can cause hypertension, arrhythmias, ventricular hypertrophy, and vascular and valvular calcification. However, it is unclear how PHPT causes hypertension. Significant hypertension with PHPT suggests possible multiple endocrine neoplasia syndrome type 2 with pheochromocytoma and requires further evaluation.

110
Q

Obstructive sleep apnea

A

Characterized by daytime sleepiness, snoring, nonrestorative sleep, gasping or choking at night, witnessed apnea, morning headaches, obesity, large neck circumference, and crowded oropharyngeal airway.

Dx: Diagnosis is established with polysomnography.

Tx: Treatment with positive airway pressure may decrease blood pressure modestly in some patients.

111
Q

Coarctation of the Aorta

A

Etiology

  • Congenital
  • Acquired (rare) (eg, Takayasu arteritis)

Clinical features

  • Upper body
    • Well developed
    • Hypertension (headaches, epistaxis)
  • Lower extremities
    • Underdeveloped
    • Claudication
  • Brachial-femoral pulse delay
  • Upper & lower extremity blood pressure differential
  • Left interscapular systolic or continuous murmur

Diagnostic studies

  • ECG: Left ventricular hypertrophy
  • Chest x-ray
    • Inferior notching of the 3rd to 8th ribs
    • “3” sign due to aortic indentation
  • Echocardiography: Diagnostic confirmation

Treatment

  • Balloon angioplasty ± stent placement
  • Surgery

Coarctation of the aorta is a narrowing of the descending aorta (typically located just distal to the origin of the left subclavian artery) that results in a proximal increase in arterial pressure and decreased blood flow to the lower body. Patients usually present with asymptomatic hypertension; however, epistaxis, headaches, and lower extremity claudication can occur. A continuous murmur is characteristically present but may be difficult to auscultate in the supine position. Patients should be initially evaluated for coarctation of the aorta with:

  • Simultaneous palpation of the brachial and femoral pulses to assess for brachial-femoral delay
  • Bilateral upper extremity (supine position) and lower extremity (prone position) blood pressure measurement to evaluate for upper and lower extremity blood pressure differential

Patients with abnormal findings should undergo diagnostic confirmation with echocardiogram.

Hx: Headache, cold feet, leg pain, reduced or absent femoral pulse, delay in femoral compared with radial pulse, murmur (continuous systolic and diastolic) heard between the scapulae, and three sign on chest radiography. More than 50% of people with coarctation also have bicuspid aortic valve.

Dx:

ECG will likely show left ventricular hypertrophy

CRX will show rib notching.

Tx: Correction of the defect should be considered if the gradient is greater than 20 mm Hg, not based on level of blood pressure. Oftentimes, even after surgical correction, patients continue to be hypertensive for years, based on permanent changes in the rennin-angiotensin system.

112
Q

📺 Monitors

A

ECG (Holter) monitoring — A Holter monitor is the preferred ambulatory ECG monitoring test for patients with daily or near daily symptoms, and for patients in whom a comprehensive assessment of all cardiac activity over a 24 to 48 hour interval is required.

Implantable loop recorder — The implantable loop recorder (ILR) is a subcutaneous monitoring device for the detection of cardiac arrhythmias ILRs are most commonly utilized in the evaluation of palpitations or syncope of undetermined etiology, particularly when symptoms are infrequent (eg, less than once per month) and/or other ambulatory monitoring has been unrevealing or inconclusive [15]. These devices, which are typically implanted in the left pectoral region and are MRI-conditional safe, store events when the device is activated automatically according to programmed criteria or triggered by the patient.

113
Q

Carotid artery stenosis

A

All patients with carotid artery stenosis should receive intensive medical therapy including an antiplatelet agent (eg, aspirin), a statin, and careful blood pressure control. Risk factors, including diabetes, obesity, and smoking, should be managed aggressively. Further management is dependent on patient symptomatology and severity of the stenosis.

  • Symptomatic carotid stenosis is defined by the occurrence of transient ischemic attack (TIA) or stroke in the distribution of the affected artery within the previous 6 months. Carotid endarterectomy (CEA) is recommended for symptomatic patients with high-grade carotid stenosis (generally 70%-99% for symptomatic lesions).
  • Asymptomatic carotid stenosis is defined as carotid atherosclerosis without recent TIA or stroke. Current guidelines differ in regard to recommendations for CEA in asymptomatic patients, although some evidence suggests benefit in those with high-grade stenosis (generally 80%-99% for asymptomatic lesions). Asymptomatic patients with lower-grade (<80%) lesions are managed medically.
114
Q

🔵Chronic venous insufficiency (CVI)

A

Chronic venous insufficiency (CVI) is most commonly caused by incompetence of venous valves leading to venous hypertension in the deep venous system of the legs. Patients may present with leg discomfort, pain, or swelling that is typically worse in the evening or following prolonged standing and improves after walking or leg elevation.

Pitting edema is the most common physical examination finding. In relatively severe cases, redirection of blood from the deep venous system to the superficial venous system may lead to other physical examination findings, including abnormal venous dilation (eg, telangiectasia, varicose veins), skin discoloration, lipodermatosclerosis, or skin ulceration (characteristically on the medial aspect of the lower leg). Risk factors for CVI include advancing age, obesity, family history, pregnancy, sedentary lifestyle, previous LE trauma, and previous LE venous thrombosis.

The diagnosis of CVI is usually based on history and physical examination

Tx: Initial treatment includes 🦵leg elevation, exercise, and 🧦 compression stockings. Patients who do not respond to initial conservative measures should undergo venous duplex ultrasound to confirm the diagnosis of CVI by identification of venous reflux (retrograde venous blood flow) in the deep venous system.

115
Q

Lemierre syndrome

A

Septic thrombosis of the internal jugular vein. The diagnosis should be suspected in anyone with pharyngitis, persistent fever, neck pain, and septic pulmonary emboli.

Dx: CT of the affected vessel with contrast would confirm the diagnosis. Treatment includes intravenous antibiotics that cover streptococci, anaerobes, and β-lactamase-producing organisms. Penicillin with a β-lactamase inhibitor and carbapenem are both reasonable choices (eg, ampicillin-sulbactam, piperacillin-tazobactam, ticarcillin-clavulanate).

116
Q

Peripheral Edema

A

Bilateral edema:

CHF (dyspnea, rales, or JVD) would necessitate a chest x-ray to rule in the diagnosis, followed by an echocardiogram.

Liver Failure: Ascities; Dx: Liver function studies are needed.

If these are absent, the clinician should check an urinalysis. If the sediment is abnormal, nephritic syndrome or acute tubular necrosis (ATN) is the likely diagnosis.

Medications:

Calcium channel blockers: Related to preferential dilation of precapillary vessels (arteriolar dilation), which leads to increased capillary hydrostatic pressure and fluid extravasation into the interstitium. Amlodipine and nifedipine are potent arteriolar dilators and cause more peripheral edema than non-dihydropyridine CCBs (eg, diltiazem, verapamil).

Direct vasodilators, β-blockers, centrally acting agents, and antisympathetics also can cause edema. Of the diabetic medications, insulin sensitizers, such as rosiglitazone often cause edema. Hormones, corticosteroids, and NSAIDs also cause problems.

Unilateral Edema:

Angioedema

Hereditary:

Dx: C4 level and C1 esterase

Drug-induced:

Urticaria

Chronic spontaneous Hx:

Venous insufficiency is the most common cause of lower extremity edema; it affects approximately 2% of the population at large. Failure of venous valves allows blood to pool in dependent areas such as the legs resulting in an increase in capillary hydrostatic pressure. This increased pressure favors increased filtration of fluid out of the capillaries into the interstitial tissue. This process causes a decrease in intravascular volume, which stimulates the kidneys to retain water and salt ultimately leading to further progression of edema.

Lymphatic obstruction is an uncommon cause of edema. It may result from malignant obstruction of lymph nodes, lymph node resection, trauma and filariasis. It classically affects the dorsa of the feet and causes marked thickening and rigidity of the skin.

117
Q

⚡ Shock

A

“Inability of the circulatory system to provide adequate tissue perfusion, which potentially leads to cellular dysfunction.”

◦ In some cases, non-focal, vague symptoms such as weakness, altered mental status, or malaise may be the only presenting signs of any of the types of shock.

118
Q

Hypovolemic shock

A

Characterized by decreased intravascular volume leading to low cardiac filling pressures, low cardiac output, and compensatory high systemic vascular resistance. Patients may have a history of obvious bleeding, from trauma or an anatomic source (GI, vaginal, or ENT), decreased PO intake or fluid loss due to vomiting, diarrhea, excess urination or other conduit (ostomy).

Most healthy subjects can tolerate the loss of 10% (500 mL) blood volume without developing symptoms or signs of shock, the body’s homeostatic mechanisms compensating effectively for the reduced intravascular volume. The loss of 20–30% of blood volume over a short period of time, however, produces shock . The reduced blood volume is partially compensated for by increased activation of homeostatic mechanisms, but the patient will have signs of shock such as tachycardia and cool peripheries. The loss of 30–40% (up to 2 L) of blood volume causes decompensated shock. The patient is hypotensive and tachycardic with signs of organ failure. This is fatal unless treated urgently and aggressively.

RA Pressure Decreased (4 mm Hg is normal)

PCWP Decreased (9 mm Hg is normal)

Cardiac Index Decreased (2.8 - 4.2 L/min/m2 is normal)

SVR Increased (1,150 dyne-sec/cm5 is normal)

MVO2 Decreased (60 - 80% is normal)

Tx:

+Calcium, warming

119
Q

Cardiogenic shock

A

Shock caused by inadequate cardiac output of any etiology (ischemia, infarction, cardiomyopathy, arrhythmia, etc.) to perfuse the tissues.

Accompanied by a compensatory increase in SVR.

LV: Patients with decreased cardiac contractility can develop cardiogenic shock due to left ventricular dysfunction. Patients generally have elevated right atrial pressures and PCWP (due to volume overload) and a compensatory elevation in SVR. The decreased cardiac output decreases tissue perfusion, which signals tissues to extract more oxygen and thereby decreases MvO2.

RA Pressure Increased (4 mm Hg is normal)

PCWP Increased (9 mm Hg is normal)

Cardiac Index Decreased (2.8 - 4.2 L/min/m2 is normal)

SVR Increased (1,150 dyne-sec/cm5 is normal)

MVO2 Decreased (60 - 80% is normal)

120
Q

Obstructive shock

A

This form of shock results from acute obstruction to flow in the circulation. Examples include impaired diastolic filling of the right ventricle (e.g., superior vena cava syndrome, obstruction of right ventricular output (e.g., massive pulmonary embolism, and an air embolus from cardiopulmonary bypass or central line placement. Systemic arterial hypertension severe enough to impair left ventricular function or acute pericardial tamponade or constrictive pericarditis can also produce an obstructive shock pattern.

Tamponade: Patients will have equalization of cardiac chamber end-diastolic pressures with increased right atrial pressure, right ventricular pressure, and PCWP (although blood flow to the left atrium is decreased, PCWP is increased due to external compression). Cardiac output is decreased, causing low MvO2.

Cardiac filling pressures are usually increased owing to outflow obstruction, impaired ventricular filling, or decreased ventricular compliance. Therefore the clinical manifestations of cardiogenic and obstructive shock may be similar.

121
Q

Anaphylactic shock

A

Results from release of inflammatory mediators (histamine, prostaglandins, cytokines, leukotrienes, etc.) resulting in systemic changes (eg, depressed myocardial contractility, vascular instability) and shock. Clinical presentation includes urticaria and/or angioedema, shortness of breath and wheezing, stridor due to laryngeal edema, pulmonary edema, and hypotension. Similar to severe sepsis, the systemic vascular resistance is typically low, and the cardiac output is elevated. Diagnosis is made when the typical signs and symptoms occur shortly after exposure to a suspected antigen and the absence of fever or infection. Treatment with epinephrine should be part of the initial management. Glucocorticoids and antihistamines are also indicated.

122
Q

Cardiac myxoma

A

Tumor characteristics

  • Benign neoplasm, 80% located in left atrium

Clinical features

  • Position-dependent mitral valve obstruction
    • Mid-diastolic murmur
    • Dyspnea, lightheadedness, syncope
  • Embolization of tumor fragments (eg, stroke)
  • Constitutional symptoms (eg, fever, weight loss)

Diagnosis & management

  • Echocardiography & prompt surgical resection

Myxomas are benign tumors and are the most common primary cardiac neoplasm; approximately 80% arise in the left atrium. Fragments of a left atrial myxoma can embolize into the systemic circulation, leading to stroke or other acute ischemia (eg, limb, mesenteric).

Left atrial myxomas frequently obstruct blood flow from the left atrium to the left ventricle, leading to a murmur mimicking that of mitral stenosis (ie, middiastolic rumble at the apex). The obstruction can also lead to decreased cardiac output that manifests as dyspnea, lightheadedness, or syncope. Because the mass is typically mobile, obstructive symptoms may be transient and influenced by position (ie, mitral obstruction is exacerbated by upright posture but is alleviated by lying down); tumor movement occasionally causes a characteristic “tumor plop” sound at the end of diastole on auscultation. In addition, some myxomas can produce cytokines (eg, IL-6) that lead to systemic inflammation (evidenced by an elevated erythrocyte sedimentation rate) and constitutional symptoms (eg, fever, weight loss).

The diagnosis is typically established by echocardiography. Prompt surgical resection is recommended to minimize the risk of embolization and sudden cardiac death (due to impaired cardiac output).

123
Q

Neurogenic (spinal) shock

A

Occurs after injury to the spinal cord or other severe CNS injury; thought to be caused by autonomic nervous system dysfunction. It is associated with low systemic vascular resistance and, typically, bradycardia. Bradycardia and hypotension in a patient with spinal cord injury should raise suspicion for neurogenic shock.

124
Q

Dx: Shock

A

Shock should be strongly considered in ill-appearing patients with vital sign abnormalities (particularly tachycardia and hypotension), altered mental status, or signs of organ hypoperfusion.

Studies should determine whether end organ damage (neurologic, cardiac, renal, GI) is present. In shock patients, the following tests should be considered:

CBC and coagulation studies (to determine anemia/blood loss, infection, hypocoagulability)
◦ electrolytes
BUN/creatinine and urinalysis; hepatic function panel (to assess liver and renal function)
chest x-ray, EKG
lactate (to gauge the degree of hypoperfusion)
◦ urine pregnancy test
◦ More invasive testing is often required: arterial blood gas for O2/pH; central venous oxygen measurement, systemic vascular resistance, and cardiac output may be measured through special central venous catheters

If a particular type of shock is suspected, further studies may be directed accordingly:
◦ infectious etiology (sepsis) – blood, sputum, urine, pelvic, or wound cultures; head CT and lumbar puncture; targeted imaging (US/CT)
◦ cardiogenic – cardiac enzymes and echocardiogram
◦ obstructive – CT or V/Q scan (PE), echo (tamponade)

125
Q

Px: Shock

A

Physical Exam findings are also variable. Blood pressure alone should not be used as the sole marker to determine shock.

Early shock may present with normal or even elevated blood pressure, and normal heart rate; but, if left untreated, tachycardia and hypotension will follow. Hypoperfused patients often exhibit cool, pale or cyanotic skin with decreased capillary refill and dry mucous membranes; confusion, altered mental status or coma; thready pulses or tachypnea. In cardiogenic shock, arrythmias, jugular venous distention, and dependent edema may be present.

126
Q

Tx: Shock

A

Before volume resuscitation is initiated in a critically ill patient, preload responsiveness should be assessed; that is, whether cardiac output is likely to improve with the administration of fluids❓.

Blood pressure will probably increase in a hypotensive patient with low CVP when a fluid bolus is given; that is, the patient will be preload responsive. Patients with low CVP who are given a fluid challenge and show at least a 2–mm Hg increase in pressure probably have intravascular depletion and are preload responsive. Such patients require additional fluid administration to optimize preload and cardiac output.

Conversely, blood pressure will NOT increase after volume infusion in a hypotensive patient with elevated CVP. Normal or elevated CVP should not be interpreted as indicating adequate circulatory volume and cardiac preload. A method other than measurement of CVP should be used to assess preload responsiveness.

127
Q

tricuspid regurgitation (TR) due to an adverse effect of his permanent pacemaker.

A

The right ventricular lead of a transvenous implantable pacemaker or cardioverter-defibrillator passes through the superior vena cava into the right atrium and then through the tricuspid valve to terminate in the endocardium of the right ventricle. Damage to the tricuspid valve leaflets or inadequate leaflet coaptation can occur, leading to severe TR in 10%-20% of patients.

Chronic, severe TR typically presents with right-sided heart failure. Patients can have distended jugular veins, pulsatile and tender hepatomegaly, abdominal distension with ascites, and lower extremity edema. Cardiac examination typically reveals right ventricular heave and a holosystolic murmur best heard at the left sternal border; the murmur intensifies with maneuvers that increase right ventricular preload (eg, deep inspiration, leg raise). The diagnosis of TR is confirmed by echocardiogram.

128
Q

Warfarin metabolism

A

CYP450
Inhibitors

↑ Warfarin effect (↑ bleeding risk)

  • Antibiotics (eg, metronidazole, macrolides)
  • Azole antifungals
  • Amiodarone
  • Cimetidine
  • Grapefruit juice

CYP450 Inducers

↓ Warfarin effect (↓ in efficacy)

  • Carbamazepine
  • Phenytoin
  • Phenobarbital
  • Rifampin
  • St. John’s wort

Acetaminophen taken at higher doses (>2 g/day) for >1 week may significantly increase the anticoagulant effects of warfarin. Although the exact mechanism is unclear, this interaction is likely mediated via enzyme inhibition in vitamin K metabolism.

129
Q

Shock index

A

SI has been shown to be a better marker for assessing severity of shock than heart rate and BP alone.

The shock index is easily calculated (heart rate divided by systolic blood pressure) and can provide clues to the severity of the patient’s condition. A normal index ranges from 0.5-0.7; repeated values >1.0 indicate decreased left ventricular function and are associated with higher mortality.

SI does not take into account the diastolic BP, and thus a modified SI (MSI) was created. MSI is defined as heart rate divided by mean arterial pressure. High MSI indicates a value of stroke volume and low systemic vascular resistance, a sign of hypodynamic circulation.

130
Q

Digitalis

A

Digitalis can increase ectopy in the atria or ventricles, which can lead to atrial tachycardia. Atrial tachycardia is distinguished from atrial flutter by its somewhat slower atrial rate (150-250 bpm as opposed to 250-350 bpm). P-waves are present, but may appear different from the p-waves normally seen when conduction originates in the SA node. In atrial tachycardia, the closer the ectopic focus is to the SA node, the closer the resemblance of its p-waves to normal p-waves originating from the SA node. In addition to causing ectopic rhythms, digitalis can also increase vagal tone and decrease conduction through the AV node, potentially causing AV block. Since it is rare for both ectopy and AV block to occur at the same time, when they do, the combination is fairly specific for digitalis toxicity.

131
Q

🎺 Beta Blockers

A

Cx: Conventional beta blockers, which affect beta-1 receptors (eg, metoprolol, atenolol) or both beta-1 and beta-2 receptors (eg, propranolol), are associated with reduced insulin sensitivity and increased risk of developing type 2 diabetes mellitus. Hemodynamic drug effects on skeletal muscle are likely a major driver of these findings.

Normally, alpha-1 receptor stimulation causes vasoconstriction, which reduces glucose uptake into skeletal muscles; beta-2 receptor stimulation opposes this effect.

In the presence of beta blockade, alpha-1‒mediated effects are unopposed, leading to further reduction in glucose uptake, higher levels of circulating glucose, and resultant increased insulin secretion from the pancreas. Insulin resistance is effectively increased (ie, insulin sensitivity is reduced). As a result, patients experience weight gain. A reduction in lipolysis in adipocytes (which is largely beta-1 receptor–mediated) likely also contributes to weight gain.

These side effects make conventional beta blockers an unfavorable option for hypertension management in patients with, or at high risk for, type 2 diabetes mellitus.

132
Q

👶🏽 CARDS 💗

A
133
Q

Tetralogy of Fallot

A

The key anomalies of this congenital heart condition include:

  • Right ventricular outflow tract (RVOT) obstruction
  • Right ventricular hypertrophy
  • Overriding aorta
  • Ventricular septal defect (VSD)

The presentation depends on the severity of the RVOT obstruction. The presence of pulmonary stenosis or atresia and infundibular spasm restricts pulmonary blood flow. In addition, exertion, feeding, and agitation can increase pulmonary vascular resistance, resulting in complete RVOT obstruction. The diversion of blood from the right ventricle into the aorta instead of the pulmonary artery results in acute hypoxemia and cyanosis (“tet” spell). Another clinical finding is a harsh, systolic ejection murmur over the left upper sternal border due to pulmonary stenosis. The single S2 comprises the normal aortic and the inaudible pulmonary components.

Tx: Acute management involves knee-chest positioning to increase systemic vascular resistance as well as inhaled oxygen to stimulate pulmonary vasodilation and systemic vasoconstriction. These mechanisms promote right ventricular blood to flow into the pulmonary artery instead of into the aorta.

134
Q

Down’s syndrome

A

Down syndrome is the most common condition caused by a chromosomal abnormality. Although prevalence correlates with increasing maternal age, children of young women can also be affected.

Neonates with Down syndrome often have low birth weight (eg, <2.5 kg [5.5 lbs]) and dysmorphic features, including upslanting palpebral fissures, epicanthic folds, and a single palmar crease.

Hypotonia is a prominent finding and can present with poor feeding due to a weak suck. On examination, an infant with weak muscle tone may have a protruding tongue. Those with low tone may slip through the examiner’s hands with arms and legs extended and floppy.

Diagnosis is confirmed by an additional copy of chromosome 21 on karyotype.

💗Cx: Complete atrioventricular septal defect (CAVSD) is the most common congenital heart defect in patients with Down syndrome. Failure of the endocardial cushions to merge results in both ventricular septal defect (VSD) and atrial septal defect (ASD) as well as a common atrioventricular valve due to poor mitral and tricuspid valve development.

In CAVSD, heart failure results from blood mixing between the chambers and severe atrioventricular valve regurgitation, leading to volume overload and excessive pulmonary blood flow. Diaphoresis/dyspnea with feeds and crackles typically manifest around age 6 weeks as pulmonary vascular resistance falls. Auscultation may reveal the following:

  • Fixed split S2 due to delayed pulmonary valve closure from flow across the ASD
  • Systolic ejection murmur from increased flow across the pulmonary valve due to left-to-right shunt across the ASD
  • Holosystolic murmur of VSD that may be soft or absent if the defect is large
  • Holosystolic apical murmur depending on the degree of AV valve regurgitation
135
Q

Coarctation of the aorta

A

Results from thickening of the tunica media near the junction of ductus arteriosus and the aortic arch. Luminal narrowing causes a mechanical obstruction to aortic blood flow, resulting in:

  • Upper extremity hypertension
  • Lower extremity hypOtension and hypoxia
  • Diminished/delayed femoral pulses
  • Systolic ejection murmur at the left interscapular area

Severe aortic narrowing makes systemic blood flow dependent on the ductus arteriosus. As the ductus begins to close (normally around day 3 of life), infants may develop heart failure with tachypnea, poor feeding, fussiness, and lethargy. Patients are also at significant risk of shock and may develop prolonged capillary refill (>3 sec), metabolic acidosis, and decreased renal perfusion (decreased urine output).

136
Q

Transposition of the great vessels (TGV)

A

Transposition of the great vessels (TGV) is the most common congenital cyanotic heart disease in the neonatal period. Abnormal rotation of the great vessels during cardiac development results in an aorta arising from the right ventricle and the pulmonary artery from the left ventricle (“arterial switch”). As a result, deoxygenated blood coming from the body goes to the right atrium and ventricle and is cycled back to the body through the aorta. Oxygenated blood from the lungs is returned to the lungs by the left side of the heart through the pulmonary artery.

Characteristic findings include cyanosis within 24 hours of life, a single S2 on auscultation (absent pulmonary component of S2 because the aorta is anterior to the pulmonary artery), and a narrow mediastinum (“egg on a string”) on x-ray.

After delivery, mixing of deoxygenated and oxygenated blood is necessary for survival and typically occurs through a patent ductus arteriosus or ventricular septal defect (both of which cause murmurs) or through a patent foramen ovale (usually no murmur). This infant most likely is mixing through a patent foramen ovale as there is no murmur on examination. If there is inadequate mixing, the infant will rapidly develop severe cyanosis and pulmonary edema and die unless a shunt is created (ie, emergency atrial septostomy). Therefore, if TGV is suspected, prostaglandins should be initiated to keep the ductus arteriosus patent to optimize inter-circulatory mixing, and echocardiography should be obtained.

137
Q

Pulmonary Function Tests

A

The four broad categories of physiologic abnormalities include:

  • Obstructive disorders (asthma and COPD)
  • Restrictive disorders of the lung (pulmonary fibrosis)
  • Restriction due to factors outside the lung (chest wall limitation due to obesity, pleural disease, or musculoskeletal disorders; weak chest wall such as Guillain-Barré syndrome)
  • Disorders resulting in impaired gas exchange with normal mechanical function (Pulmonary embolism)
  • Mixed

Poor preformance: irregular flow-volume curve or poor reproducibility.

Normal flow-volume curve is roughly triangular

A tracheal plateau, which is a normal variant usually seen in younger subjects, is caused by normal flow limitation in the trachea in the absence of peripheral airway obstruction.

Patients with obstructive disorders typically have a reduced FEV1/FVC ratio and a flow-volume curve with a “scooped out” appearance

The next step is to deterdermine whether the obstruction can be reversed by the administration of a bronchodilator.

In obstructive diseases, the degree of impairment of pulmonary function can be classified on the basis of the FEV1. According to one commonly used scale, an FEV1 less than the lower limit of normal but greater than 70% is mild, 60 to 69% is moderate, 50 to 59% is moderately severe, 35 to 49% is severe, and less than 35% is very severe.

Bronchoprovocation studies, in which graded doses of a stimulus are used to elicit airway constriction, are performed to measure airway responsiveness. A responsive airway, that is, one in which a small stimulus leads to a fall in FEV1, may be used to confirm the diagnosis of asthma

Patients with restrictive disorders have reduced lung volumes and typically have a flow-volume curve with a “witch’s hat” shape—a tall peaked curve.

Restriction may be due to either reduced lung compliance or mechanical changes to the chest wall and tissues surrounding the lungs (e.g., obesity, muscle weakness, chest wall deformity, pregnancy, pleural effusion, or heart failure)..

For many restrictive diseases, the severity of the restriction can be graded with use of the total lung capacity as a percentage of the predicted value.

In patients with restriction caused by interstitial disease, the total lung capacity and the vital capacity or FVC are usually reduced by a similar proportion.

In some patients with restriction, the total lung capacity as a percentage of predicted and the vital capacity percentage of predicted are quite different (>10% difference).

Some patients have a mixed disorder with evidence of both obstruction and restriction. Common causes include cystic fibrosis, sarcoidosis, and heart failure as well as cases in which the causes of the obstructive disorder and the restrictive disorder are unrelated.

Disorders of the central airways can cause characteristic patterns of abnormality:

In a “fixed airway obstruction” such as tracheal stenosis, flow is typically reduced on both inspiration and expiration.

in a variable extrathoracic (upper) airway obstruction, inspiration is disproportionately reduced; however, expiration is often abnormal, merely less so.

Likewise, in variable intrathoracic obstruction (e.g., relapsing polychondritis, tracheomalacia, or a dynamic intrathoracic tracheal tumor), the expiratory flow-volume curve is reduced but in a pattern unlike that seen in asthma or COPD

In patients with heart disease, a decline in FEV1/FVC ratio is associated with underfilling of the left heart and low cardiac output. By comparison, a decline in FVC with preserved FEV1/FVC ratio is associated with left ventricular hypertrophy and diastolic dysfunction.

The effects of obesity on lung function are usually relatively modest among ambulatory patients with a body mass index (BMI) less than 40. The most commonly observed effect of obesity on lung function is a reduction in expiratory reserve volume (the amount of air exhaled between FRC and residual volume), which is substantially reduced even in persons who are overweight (BMI 25 to 30) or mildly obese (BMI 30 to 35). Vital capacity is reduced in obesity, but the effect is modest, usually within the normal range, and highly variable. In large studies, vital capacity or FVC is reduced on average by 0.5 to 0.8% for each unit increase in BMI above 25. Effects of obesity on total lung capacity and FEV1 are somewhat smaller. The FEV1/FVC ratio and Dlco actually increase slightly with increasing BMI.

DLCO - 60 40 Mild, moderate, severe

138
Q

BAL

A

The differential cell count on a normal bronchoalveolar lavage specimen includes 85% macrophages or more, 10 to 15% lymphocytes, 3% neutrophils or less, 1% eosinophils or less, 1% mast cells or less, and less than 5% squamous epithelial cells (which are an indicator of contamination from the upper airway). Smokers may have higher cell counts and a higher percentage of neutrophils. Increased lymphocyte counts are seen in sarcoidosis (Chapter 89), hypersensitivity pneumonitis (Chapter 88), nonspecific interstitial pneumonitis (Chapter 86), collagen vascular diseases (Chapter 86), radiation pneumonitis (Chapter 88), cryptogenic organizing pneumonia (Chapter 86), and lymphoproliferative disorders. Increased neutrophil counts are seen in idiopathic pulmonary fibrosis (Chapter 86), collagen vascular diseases (Chapter 86), infectious pneumonia (Chapter 91), aspiration pneumonia (Chapter 91), acute respiratory distress syndrome (Chapter 96), diffuse alveolar damage (Chapter 85), acute interstitial pneumonia (Chapter 86), and asbestosis (Chapter 87). Increased eosinophils can be seen in asthma (Chapter 81), bronchitis (Chapter 90), allergic bronchopulmonary aspergillosis (Chapter 319), eosinophilic granulomatosis with polyangiitis (Chapter 254), Hodgkin lymphoma (Chapter 177), and drug-induced lung disease (Chapter 88). If eosinophils are more than 25%, eosinophilic pneumonia is likely (Chapter 161). If lymphocytes are increased and the clinical differential diagnosis includes sarcoidosis or hypersensitivity pneumonitis, analysis of T-cell populations may be helpful; the CD4:CD8 ratio is typically increased in sarcoidosis but reduced in hypersensitivity pneumonitis. If more than 20% of macrophages stain positive for hemosiderin, diffuse alveolar hemorrhage is considered likely (Chapter 85), particularly if lavage fluid is progressively bloody in successive aliquots of
lavage fluid.

Cellular constituents of bronchoalveolar lavage are usually stained for cytologic analysis for malignant cells and viral inclusions. If Langerhans cell histiocytosis (Chapter 86) is considered possible, 5% or more CD1a–positive cells support the diagnosis. If chronic beryllium disease or beryllium sensitization is possible, a lymphocyte proliferation test in response to exposure to beryllium salts can be helpful (Chapter 87). Staining of solid material from the bronchoalveolar lavage with periodic acid–Schiff (PAS) stain for the presence of PAS-positive material is essential to the diagnosis of pulmonary alveolar proteinosis (Chapter 85). A diagnosis of lipoid pneumonia (Chapter 88), caused by the aspiration of oil, can be confirmed by an excess of lipid-laden macrophages from bronchoalveolar lavage. The presence of asbestos bodies or silica is not diagnostic of lung disease related to these substances (Chapter 87) but does indicate significant exposure.